[ 3 / biz / cgl / ck / diy / fa / ic / jp / lit / sci / vr / vt ] [ index / top / reports ] [ become a patron ] [ status ]
2023-11: Warosu is now out of extended maintenance.

/diy/ - Do It Yourself


View post   

File: 192 KB, 1200x1200, 0009324602_10[1].jpg [View same] [iqdb] [saucenao] [google]
1498817 No.1498817 [Reply] [Original]

faulty thread: >>1495036

>RULES
0. Electrics ≠ electronics. Mains wiring goes to /qtddtot/ or /sqt/. PC assembly to >>>/g/.
1. Know the law. You are assumed to understand Ohm's Law and Kirchhoff's circuit laws. If not, google.
2. RTFD. Re-read all documentation/datasheets pertinent to your components/circuits before you ask.
3. Pics > 1000 words. Post schematic/picture/sketch/9001.5 hours in MS Paint with all part numbers/values/etc. when asking for help. Focus/lighting counts.
4. Read posts fully before replying.

>I'm new to electronics, where to get started?
It is an art/science of applying principles to requirements. Find problem, learn principles, design and verify solution, build, test, post results, repeat.

>Project ideas:
http://adafruit.com
http://instructables.com/tag/type-id/category-technology/
http://makezine.com/category/electronics/

>Principles (by increasing skill level):
Mims III, Getting Started in Electronics
Platt, Make: Electronics
Geier, How to Diagnose & Fix Everything Electronic
Kybett & Boysen, All New Electronics Self-Teaching Guide
Scherz & Monk, Practical Electronics for Inventors
Horowitz and Hill, The Art of Electronics

>Design/verification tools:
LTSpice
falstad.com/circuit/circuitjs.html
NI Multisim
CircuitLab
iCircuit for Macs
KiCAD (pcb layout software, v5+ recommended)

>Components/equipment:
Mouser, Digi-Key, Arrow, Newark, LCSC (global)
RS Components (Europe)
eBay/AliExpress sellers, especially good for component assortments/sample kits (caveat emptor)
Your local independent retail electronics distributors
ladyada.net/library/procure/hobbyist.html

>Related YouTube channels:
mjlorton
paceworldwide
jkgamm041
eevblog
EcProjects
greatscottlab
AfroTechMods
Photonvids
sdgelectronics
TheSignalPathBlog
BigClive

>Li+/LiPo batteries
Read this exemplary resource first: https://www.robotshop.com/media/files/pdf/hyperion-g5-50c-3s-1100mah-lipo-battery-User-Guide.pdf
>I have junk, what do?
Take it to the recycler.

>> No.1498831

Going to make my first PCB using toner transfer. What is the smallest route width you think one can use for this method without having problems?

>> No.1498838

I struggle to understand how connecting two non-floating power supplies together is causing a short, any ideas?

>> No.1498840

>>1498831
varies with the exact "equipment" you're using, but it's not unreasonable to get down to around .15mm traces and around the same clearance. I would do a test board with a range of trace thicknesses starting at 0.1mm to get an idea of your capabilities.

>> No.1498846

>>1498838
Connecting how and floating how?

>> No.1498847

>>1498846
Well basically I was told that if you have for example two computer power supplies and you use one of them to power some DC device A, and another one (psu) to power some simple circuit which supplies PWM to this aforementioned device A, you have to make sure one of the supplies is floating otherwise the whole thing will go up in flames and i can't understand why that is.

>> No.1498850
File: 40 KB, 1516x808, Untitled.png [View same] [iqdb] [saucenao] [google]
1498850

>>1498847

>> No.1498853
File: 28 KB, 1127x1597, psu.png [View same] [iqdb] [saucenao] [google]
1498853

>>1498850
You can hook two PSUs in series no problem but when you're putting them in parallel you wanna match the voltages as close as possible. Since it's usually impossible to get them exactly the same and to maintain it with variations in load the general way this is handled is to put either put small value resistors between each PSU and the load or to put diodes there instead. Generally resistors should probably be in the range of 10 ohms to a few hundred ohms though exactly how you size them depends on your maximum load current and how much power you're willing to dissipate in said resistors. For the diode arrangement you probably wanna use Schottky diodes for the low forward voltage drop which will minimize power dissipation that is unless relatively high reverse current in the milliamp range is an issue for your PSU for whatever reason.

>> No.1498860
File: 105 KB, 600x315, PC817.png [View same] [iqdb] [saucenao] [google]
1498860

>>1498817
Ritual

>> No.1498861

>>1498853
>>1498850
oh i see, so when the current arrives at a fork in the circuit, it will go in to the second psu because it is now creating a low resistance path to the common ground, which is the outlet earth pin to which both of the supplies are connected and if one is floating, then there is no potential between the + output on the psu 1 and ground on psu 2
so basically slapping a diode on every of the 4 wires coming from the two psus i am using (+/- +/-) will eliminate this problem

>> No.1498873

If a resistor gets hot, does its resistance usually drop or increase?
I assume it drops as the thermal energy would create more free charge carriers?

>> No.1498876

>>1498873
In most materials resistance increases with heat.
In some it decreases within a certain window. I think we did an experiment about this in school with graphite from a pencil.

>> No.1498881

How can I turn on something that uses 10 A or more without passing all through the ON button?

>> No.1498882

>>1498881
you can let the on button control a relay

>> No.1498884

>>1498798
Nice, I’m gonna hook up a shit ton of batteries to the solar panels and use them as a grow light. What’s better for plants than sunlight?

>> No.1498892

>>1498881
You can use relay but the clicking will drive you insane. A better solution would be to use a mosfet which is completely silent, costs less than a relay, and even let's you turn whatever you want to turn on and off thousands of times per second

>> No.1498895

>>1498892
But how I`ll actvate the mosfet with a button? Just hook one of the button to VCC the other to the gate and let the gate be pulldown?

>> No.1498899
File: 306 KB, 597x366, mfw.png [View same] [iqdb] [saucenao] [google]
1498899

how do i use a transistor to amplify voltages under Vbe of the transistor
do i add a resistor to base so the voltage there is just under Vbe and the input gets added to it so it starts conducting

>> No.1498903
File: 37 KB, 272x245, eve dog.jpg [View same] [iqdb] [saucenao] [google]
1498903

>have some old rectfier bridges
>using them in my first PCB
>of course there is no foot print
>the NI utilboard guide to create new components is the size of your mom
>too cheap to buy new ones with the proper current rating
>tempted to just half ass 5 1n4007 discrete diode bridges and call it a day
>no time to learn how to draw and make foot prints
I have only myself to blame for this hell

>> No.1498910

>>1498895
you have to add a 10k resistor as a pull down between gate and ground

>> No.1498914

>>1498903
If Im in that situation I just manually place some 1-pin headers.

>> No.1498919

I have a question. I want to plug 4 speakers to an amplifier. they all are 6ohm. what do I have to get? a 6 ohm amplifier? bigger? smaller?

>> No.1498921

>>1498919
you need to get an amplifier with a 6/4 impedance, or AKA 1.5 Ohm. Or you correct the impedance adding the extra 4.5 Ohms in series with the load.

>> No.1498942
File: 28 KB, 871x789, series power supplies.gif [View same] [iqdb] [saucenao] [google]
1498942

>>1498847

in your situation, you're probably ok. the danger is when you try to put non-floating supplies in series. see pic.

in the top part, the supplies are floating, so the output voltage just adds. in the lower part, all the supplies are grounded at the plug (shown in green). the case is grounded, and the (-) output is grounded. so, when put them in series, you're effectively shorting out the 2 lower supplies (shown in orange).

>> No.1498961
File: 88 KB, 1920x1053, file.png [View same] [iqdb] [saucenao] [google]
1498961

>>1498899
High-pass filter to a bias voltage. Make sure your signal frequency is above the cutoff frequency.

>> No.1498963

>>1498942
so basically all i need to do is snip the green wire going into the psu then?

>> No.1499014
File: 5 KB, 259x194, ge.jpg [View same] [iqdb] [saucenao] [google]
1499014

>>1498963
Use a ground eleminator plug.

>> No.1499017

>>1498963

nope. you must also make sure the metal cases never touch, and no metal objects touch both at the same time.

>> No.1499028
File: 64 KB, 612x612, 1496722061932.jpg [View same] [iqdb] [saucenao] [google]
1499028

>>1498817
Elec eng here. The circuit design courses that were offered at my school were abysmal and mostly were only offered if you were doing a masters.
I ended up landing a job as a mech design engineer, but I still want to get a hardware design engineer where I design circuits.
Nothing I can really do to get my foot into the hardware design field besides getting my masters or doing some hardware design project in my garage. I probably won't do more schooling since school hasn't gotten me anywhere in terms of job opportunities and I'm sick of reading theory with no real world context to apply it all to, so I'll just build something worthwhile.
What are some projects that will wow employers/give me some worthy knowledge/experience in circuit design so I can at least bullshit my way through an interview? I'm thinking a dev board, but I'm afraid that might pigeonhole me into an embedded software position.

>> No.1499030

Would it be plausible to draw 4A from 4 AA lithium batteries to power a motor(s)? What kind of voltage sag would I be looking at? If I were to save the lithium and use NiMH for testing, how would they cope?

>> No.1499034

>>1499030
Actually, expanding on this, is there a way to determine what kind of current a motor will inherently draw without testing it? There isn't exactly good data available for most motors and I think it entirely possible that a performance motor could just kill aa batteries. Application is an rc car project with 4 aa batteries as a constraint.

>> No.1499042

>>1499034
Are you hooking up the AA batteries in series or parallel? If series, you have a zero percent chance of getting that current; the internal resistance of the batteries is too much. If you are hooking them up in parallel so you have 1.5 volts output, you may be able to get that current for 10-20 seconds.

Why not just use 18650 lithium ion batteries? They are slightly larger than AA, but they can produce a couple of amps for an hour. If you made a 2S2P lithium ion battery pack, it would output about the same voltage as 4 AA in series, and would be good for maybe up to 4A.

>> No.1499045

>>1499028
Many new grads are in your position, with no circuit design experience. As such, doing just something will often set you apart from many peers. I think you'd struggle to find a project with the wow factor unless you include at least some embedded element, unless you go for maybe an RF or highspeed front end for 'something'. If you really have little circuit design experience, consider something which you can start off dead simple and add more and more features to. A bench top power supply might be good as you can start with just a couple of regulators, then making adjustable outputs, then adding volt and current measurements, then digital control, then a serial interface to a PC, then designing your own feedback circuitry, etc. Best of luck.

>> No.1499046

>>1499042
Restricted to AA batteries, unfortunately. I'm trying to figure out the maximum power achievable under that restriction and whether its worth getting a real motor. I forgot to consider the batteries needing to be in parallel for additive current, though I've seen indication that lithium batteries are supposed to be able to run as high as 1A with reasonable lifetime. Are such figures meant to represent cumulative momentary peak loads as in a camera flash?

The question comes down to how much power can I pull from the batteries, and will that be enough for the motor at the gear ratio selected. The motor will need all the voltage, as I understand, meaning I need the full 6V, so what is the maximum continuous (with 10-30s running time) current I can pull from the best AA batteries (apparently lithium) available?

>> No.1499047
File: 57 KB, 1702x699, Untitled.png [View same] [iqdb] [saucenao] [google]
1499047

>>1499028
I`m at the same place buddy, 6th semester here. Started making my first shit today, a power supply (I think I`ll add a sine wave generator if my scope arrives before I get to make the pcb.). never messed with board design before. No idea if telling I made stuff with barely no tools and just my thick head and tears will get me any jobs but I like it.

>> No.1499048

>>1499042
>>1499046
Actually now I'm reading that NiMH batteries, which I have, have much lower internal resistance and can do considerably higher current. Would a pack of four of those in series, in your experience, do the job?

>> No.1499051

>>1499046
>maximum continuous current that I can get from 4 AAs?

I would guess somewhere around 1-2A, and that would only last for a handful of 30 second trials before you are down to well under an amp.

There are some battery datasheets out there in Google that you can use to get a better approximation. AAs are a major limiting factor though.

Also, don't forget, pushing lithium batteries too far will cause them to ignite. Keep a long delay in between each cycle, so the batteries and electronics have a chance to cool down.

>> No.1499053

>>1499048
That would probavly do it. People have been using NiMH batteries for RC applications for decades now, so that would probably be the best place to start.

Also, check how many mAH your batteries are rated for, so you can estimate how long they will last. A 1000mAH battery can provide 4 amps for 15 minutes (in theory, assuming they are perfect batteries and can output that much instantaneously)

>> No.1499062

>>1499053
Final question: How well will a 2s brushless motor run at lower voltage? 7.4V is expected and it may get as little as 4V after voltage sag.

>> No.1499066

>>1498817
If I wanted to make software or hardware that would allow me to show windows and files on an oscilloscope that would look a but like OP's pic, would it be any kind of easy? Vector graphics is one thing, but a vector monitor sounds damn cool.

>> No.1499067
File: 47 KB, 1876x1168, slowly but surely.png [View same] [iqdb] [saucenao] [google]
1499067

>>1499047
It`s like a puzzle, but fun and harder. Jesus christ, one sided boards are hard. I`ve managed to route everything without jumpers, lets see how I do for my first one.

>> No.1499071

>>1499067
nice

>> No.1499072
File: 245 KB, 1920x1080, Untitled.png [View same] [iqdb] [saucenao] [google]
1499072

>>1499071
fug forgot my pic

>> No.1499073

i saw this circuit once but i don't remember how it was made.
it was really simple,

basically there were a bunch of leds and resistors placed on a board, and two wires were sticking out. you showed them into an outlet and depending on the voltage a number of leds lit up, so it it was say 100v it was first 3 leds, if it was 230v, it was 4 leds, and basically it worked in this spirit
how would a circuit like this looked like?

>> No.1499080

>>1499073
Perhaps a resistor ladder with LEDs coming from each branch to ground?

>> No.1499090
File: 78 KB, 718x495, 4RpJ0.png [View same] [iqdb] [saucenao] [google]
1499090

>>1499080
Yeah, probably this, if it was super simple. Use can use zeners if you want it to be somewhat more accurate.

Pic related for your ultra high performance needs.

>> No.1499110

>>1499090
that chip is for 12v input, i am pretty sure 230v ac would kill it instantly

>> No.1499120

>>1499045
>>1499047
I mean I have some stuff under my belt. I built a circuit to drive and control a robotic arm in school, and I built a reflow oven controller in my garage a few months ago with my brother.
I'd love to do some RF design project. That seems to be a field where pcb traces and component arrangement can be very critical. Seems to be the most in demand job in terms of hardware design too.
A power supply does seem like something I could do, but I dunno, little motivation to make one when I already have a quality benchtop power supply in my garage. Like what's the point if I'm not gonna use it?

>> No.1499121

>>1499120
RF is really good fun, but can involve using a lot of expensive software and iterations of expensive designs, tested on expensive equipment if you are looking at certain projects. If you already have a quality bench supply, why not consider an arbitrary function generator using DDS? Involves a bit of high frequency design though requires more embedded work than a bench supply. Remember the point is to show off to a potential employee that you can design and make a project, more than you needing to use it - unless you are wanting that too...

>> No.1499122

>>1498895
pretty much, but be careful to leave margin between your top voltage and the Vgs(max) of the MOSFET. add resistance in series with the switch to adjust

>>1498899
feed the input into the emitter
see also: common-base amplifier

>>1498919
connect two series strings in parallel for (6/2)*2 = a 6 ohm load

>>1498921
lurk moar

>>1499066
hardware might be kind of easy with a fast enough x-y scope
software would be a pain in the ass

>>1499067
>one-sided
y tho

>> No.1499124
File: 273 KB, 2048x2043, really makes ur think.jpg [View same] [iqdb] [saucenao] [google]
1499124

>>1499120
>>1499121

6th semester guy here, you can make a sinusoid generator using a triangle wave source and matched transistors in a diferential configuration. The drain/collector current is hipertangential, and to the first order and small signals it`s pretty much a sine wave. I say this because DDS chips are rare-ish and old arround here. Or you could just do the quick and dirty ROM business.
>>1499122
>one-sided
>y tho
Poor third worlder and the mail service makes it expensive to buy chinkshit.

>> No.1499125

>>1499124
Yeah I'm talking about using a uC or even an FPGA to control a DAC, not just using an existing DDS IC.

>> No.1499127

>>1499125
Is there an FPGA programming software that isn`t absolute ass bleeding? My only experience was with Quartus, and for the price of those boards they really should improve their UI.

>> No.1499130

>>1499122
>software would be a pain in the ass
that's what I was thinking

>> No.1499131

>>1499127
In short, no. The stuff from the big vendors like Xilinx and Altera (Now Intel) is really just a gui for hundreds to thousands of command line programs mostly written in the 90s. As such they are all a complete mess because of legacy compatibility and because people are forced to use it so there's little incentive to improve it. If you are interested in FPGAs, I'd recommend a dev board from ebay - I used a board which had "designed by WXEDA" on it. Just search "Cyclone IV dev board" and it should cost around $30. There are others which are more basic and cost less, but you chose what suits you.

>> No.1499140

>>1499131
>getting into FPGA
no thanks, my ass is ok without bleeding of anger.

>> No.1499152

>>1499124
>expensive to buy chinkshit
ah. if I were you, I would worry more about conserving pc board space. tighten up the layout. 1/4W resistors are perfectly fine 0.1" apart. also:
>tighten up that space between your IC and the five resistors R3..R12
>don't make traces too much thinner than they need to be
>don't go to great lengths to avoid a wire jumper
>don't be afraid to swap units e.g. exchange U1a and U1b for a less tangled layout (you can see this better on the rat's nest)
>turn your "1 ohm" resistor counterclockwise
>avoid acute angles e.g. just to the right of R3, they can overetch (especially with thin traces)
>where are your grounds?

>>1499127
long answer: there's iceSTORM which has no UI, and Lattice's own iCEcube is quirky but at least not the usual Synopsys clone
I agree, the learning curve can be steep, but for the most part I just edit/pre-simulate outside their UI and only use the build functions within it
short answer: no

>> No.1499164

>>1499073
>>1499080
>>1499090
Zeners are going to be the easiest way

>> No.1499165

>>1499131
Ever heard of Nandland Go Board? I have a friend who likes it

>> No.1499166

>>1499164
but wouldn't you need zeners of all these fine divisions (247V, 242V, 237V, etc.)?

>> No.1499170

>>1499166
He's describing a tester with 4 LEDs. Unless it's binary you only need 8 zeners.

>> No.1499183

>>1499170
My point is it might be hard to get such obscure values for zeners, and such might need to combine a few smaller values.

>> No.1499184

>>1499183
It sounds like he's trying to make a tester that tests what AC voltage an outlet is on. There aren't a lot of possibilities. He won't need need a 4.238V zener

>> No.1499189

>>1499184
Well zeners come in standard values that won't necessarily give you nice 250, 240, etc. voltages.

>> No.1499201

>>1499189
You can make them easily enough. There's plenty of 110, 120, and 120V Zeners on Digikey.

It's much simpler to string together a couple of zeners, a resistor, and an LED, than to use an IC and attenuate AC voltage to a range that it'll expect, and remember how to decode the output.

>> No.1499202

>>1499189

you can put zeners in series and they add. 100V in series with 47V in series with 5.1V gives you a 152.1V zener.

>> No.1499237

>>1499202
Definitively, the problem is low voltage zeners have a very rounded cutoff and I'm wondering if an LED's forward voltage is flexible enough to cause ambiguity because of it.

>> No.1499289
File: 77 KB, 1000x1000, ea613239-37b8-4c08-8e79-cb55966dacc3.jpg [View same] [iqdb] [saucenao] [google]
1499289

What are your thoughts on this chinesium soldering stations? Do you recommend them for hobbyist use?

>> No.1499298 [DELETED] 

>>1499289
They don't look bad at all, from what I've heard. Though personally I'd go for something with higher [heat capacity].

>> No.1499361

>>1499080
so say you have two resistors once to light up 110v and one to 230v led, (and a diode to snip the negative sine)
if you stick it into an 110v outlet all is fien and dandy, 110v resistor led goes up 230v one does not. but if you stick it into a 230v outlet, 230v resistor led goes up and 110v resistor ¨led explodes because the resistor has only exactly enough resistance to light up the led at 110v

>> No.1499363
File: 39 KB, 600x600, 6031-sda-40-p134[1].jpg [View same] [iqdb] [saucenao] [google]
1499363

>>1499289
i just use this, had zero problems with it

>> No.1499369

>>1499361
Can we get avalanche LEDs or light emitting zeners?

>> No.1499371

>>1499237
LEDs aren't the greatest zeners ever. On the other hand, this 4 LED thing does not sound like super precise instrument to begin with, so maybe it is ok.

>> No.1499378

>>1499371
A cheap MCU with an ADC would probably be the easiest and a pretty cheap way to go. Could even run it to a 7-seg display for not too much more if you wanted to.

>> No.1499396

are there any decent water proof electronics cases on ali for cheap? i need to fit raspberry pi sized electronics in such cases
they don't have to bee water tight, but have to handle being outdoors in rain and shit

>> No.1499403

>>1499396
y u no like
https://www.aliexpress.com/item/115-90-55mm-Plastic-Waterproof-Electronic-Benis-Box-Chastity-Device-Sissy-Pink/32814424205.html
for $5.49, or if you don't need much room for USB
https://www.aliexpress.com/item/115-90-55mm-Plastic-Waterproof-Electronic-Benis-Box-With-Ears-Rabbit-Bachelorette-Funny/32844047005.html

>> No.1499404

>>1499396
also y u no use pi zero

>> No.1499410

>>1499404
i won¨t be using pi at all i am only saying it will be of similar size
also pi zero a weak shit

>> No.1499450

So if i charge a cap fully at a certain voltage, how do i calculate how long it can supply current i need?
for example, let's say i have 5V battery that supplies 200ma, and i connect a cap in parallel with it, and the cap charges to the 5V. how do i calculate the capacitance i need to buy in order fo this cap to be able to supply 200ma at 5V for say 1 second?

>> No.1499453
File: 60 KB, 1920x1080, Untitled.png [View same] [iqdb] [saucenao] [google]
1499453

>>1499450
I= dQ/dT and Q=CV
>>1499152
>>where are your grounds?
Power plane
>turn your "1 ohm" resistor counterclockwise
>avoid acute angles e.g. just to the right of R3, they can overetch (especially with thin traces)
Thanks
>>don't be afraid to swap units e.g. exchange U1a and U1b for a less tangled layout (you can see this better on the rat's nest)
I hoper ultiboard has an option for that, foward annotation from the circuit schematic will be boring.
>>tighten up that space between your IC and the five resistors R3..R12
>don't make traces too much thinner than they need to be
>don't go to great lengths to avoid a wire jumper
thanks

>> No.1499456

>>1499289
Cheapo T12 stations are great.

>> No.1499457

>>1499450
That is not possible, no matter how large the cap you discharge is. If you accept the usual 5V-5% = 4.75V you can use ∆V=I*t/C or C=0.2A*1s/0.25V=0.8F. Quite large.

>> No.1499460

>>1499453
Also I`m keeping things a tad spaced because I`ll be drilling with a 1mm hand drill, nothing used for pcbs..

>> No.1499470

>>1499457
what if there is some voltage drop leeway? as long as the the voltage is above 4v it should be fine as long as it can pump out the 200ma. the actual 1s was just an example, the real use will be to feed a circuit while a relay switches power from NO to NC contacts, which should be much shorter than 1s

>> No.1499477

>>1499470
You have all you need to calculate your own scenario.

>> No.1499478

>>1499378
That defeats the purpose. Simplicity is part of the OP. He's trying to make a tester that'll say which voltage an outlet is at out of, what, three or four potential values? This is a project that should take 5 minutes to design and find parts for. Zeners, LED, resistor x4. You're massively overcomplicating it.

>>1499073
What voltages do you need to distinguish between?

>> No.1499479

>>1499470
Mouthbreather math:
The voltage of a 1 farad capacitor drops one volt per second at 1A load.
Thus, 0.2F capacitor's voltage drops 1V/s at 0.2A load.
So, if you charge a 0.2F capacitor to 5V, its voltage drops to 4V after one second, if the load current is 200mA.

Relay's datasheet should list the operate and release times. 10ms would be quite typical to a small relay.

>> No.1499482

>>1499479
Perfect, looks like i don't need a huge one, but i will use twice as much just to be safe

>> No.1499499

>>1499478
>>What voltages do you need to distinguish between?
ac
110v
230v
400v
where all of them can have offsets +/- 20V

>> No.1499533

>>1499499
So Vz<90V, 130<Vz<210V, and 250<Vz<380V diode drop?

What's your use case? I'm betting you can tolerate being a few volts off.

>> No.1499542

>>1499533
recognizing mains outlet voltage

>> No.1499544

>>1499533
so if you stick it into 230V both 110V and 230V diodes light up but not the 400V, if you stick it into 400V all 3 leds light up

>> No.1499549

>>1499542
Alright, you probably won't be able to use the zener, resistor, LED solution because you'll draw too much current in the 110V branch when the input gets up to 400V. Microcontroller is likely the easiest way after all.

>> No.1499552

>>1499544
ye

>> No.1499566

>>1499549
which one though? they work in small dc ranges as far as i know, so sticking one into 400v ac will fry it instantly
and i don't want to use some insane complicated circuitry to step down the voltage and stuff
i remember that when i saw that project i was surprised how simple it was
fuck if i only could remember

>> No.1499586

>>1499566
Resistive attenuator and an active peak detect should work fine.

>> No.1499595
File: 11 KB, 330x309, VI curve for a given neon.png [View same] [iqdb] [saucenao] [google]
1499595

>>1499566

you might try series neon bulbs. an NE-2, for example, is similar to a 90V zener. (check specs for your model). so one neon = 90V or more, two in series is 180V or more, etc.

you choose series resistors that are big enough so they cover the entire expected voltage range without going over rated current. this is much easier to do than with LEDs coz lamps need very little current.

>> No.1499596
File: 31 KB, 360x360, neon voltage indicator.jpg [View same] [iqdb] [saucenao] [google]
1499596

>>1499595

found one

>> No.1499615
File: 25 KB, 675x816, MS8922A.jpg [View same] [iqdb] [saucenao] [google]
1499615

You do not tolerate junk for work in a CAT III environment
Safe voltage testers are way too cheap to diy

>> No.1499642
File: 3.31 MB, 2770x2233, 11F93C9F-D73D-4781-8539-531675EE19E1.jpg [View same] [iqdb] [saucenao] [google]
1499642

Turns out these little panels do work but a flourscent light and even strong flashlights are a shitty substitute for the sun. They get over 5V with a good angle, and that was one I didn’t polish 100% yet.

Anyway what do you guys think about this for an idiot’s first power supply?

https://www.amazon.com/Tekpower-TP3005T-Variable-Linear-Alligator/dp/B00ZBCLJSY/ref=mp_s_a_1_4?ie=UTF8&qid=1542566198&sr=8-4&pi=AC_SX236_SY340_QL65&keywords=dc+power+supply&dpPl=1&dpID=51EkvryutDL&ref=plSrch

>> No.1499682
File: 13 KB, 342x342, fine coarse.jpg [View same] [iqdb] [saucenao] [google]
1499682

>>1499642

you can prob find an equivalent unit with fine+coarse control of voltage and current. it'll mean you'll destroy fewer parts when you accidentally over-volt them.

>> No.1499700

>>1499642
I would buy it. Linear lasts forever and doesn't pollute the place with EMI. Be sure to get used to the setup procedure (push-turn..).

>> No.1499707

>>1499682
I believe on the one I posted, you push in on the dial to select which digits to change so you can go in 0.1 increments or 0.001.

Two more questions- will 5A be enough for messing around with small electronics, or is it worth spending the extra $20 for the 10A? Also there is a “Linear” and “Switching” and I think it’s the linear which tends to be more expensive. People were saying the switching creates more noise. Is it really a big advantage to go for the linear if I’m not working on sophisticated stuff?

>> No.1499708

>>1499700
Oh well that answers my question on linear vs switching. Do switching units tend to fry faster?

That was “Amazon’s Choice” and the best deal and rating for a linear supply. However there was a 10A switching with good reviews for about the same price so I’m wondering if the extra current would be a better choice over the linear supply,

>> No.1499709

DIY ways to make PCBs with a INKJET printer?

>> No.1499710

>>1499566
This guy
>>1499615
is also correct. I wasn't thinking about it until that post but I certainly wouldn't fuck around with 120VAC in any of my projects. 400V is a big "hell no"

>> No.1499716
File: 221 KB, 988x1390, nancy boy.jpg [View same] [iqdb] [saucenao] [google]
1499716

>>1499710
>I certainly wouldn't fuck around with 120VAC

of course you wouldnt, you might get a run in your stockings.

>> No.1499719
File: 480 KB, 2048x1536, IMG_20181118_212127.jpg [View same] [iqdb] [saucenao] [google]
1499719

Yay.
My hydroponics setup is finally done and fully wired. I even took the time to do cable management on the side pipe.
The duct tape bag contains a power adapter innards which convert 230v to 5v, which then powers the arduino and fans on the grow lights.
The arduino is placed in the red case along with an rtc and two relays, one of which toggles 230V power for the leds and the other one toggles 5V power for the fans.
The leds are taped to old cpu heatsinks.
The arduino works as a timer to turn on and off the lights and fans. I will later also maybe add a temp probe.
I will also probably add a piece of string and tie the the heat sinks to the top red paper pipe as a safety measure, since the heat sink mounts are 3d printed from pla and if the heat sink gets too hot they will melt and the 230V led will fall down on the hydro container with the plants which is full of water and that could be dangerous.
Overall i am really happy with it, it turned out better that is originally thought it will.

>> No.1499739

>>1499716
Let me know when you start college, electrician

>> No.1499757

>>1499478
Just saying, the MCU method will take less parts than the 6 or 7 zeners.

But at ±20V zeners will do the job just fine. I was assuming he wanted to tell the difference between 220V, 230V, and 240V.

Using voltage dividers and MOSFETs (as shitty comparators) could be an OK way to go, pic related. The dropper resistor is burning 10W though, so either drop the LED current from 20mA or use a capacitive dropper+zener to get a lower DC rail.

>> No.1499768
File: 62 KB, 666x452, latest[1].jpg [View same] [iqdb] [saucenao] [google]
1499768

>>1499757

>> No.1499771

>>1499708
switching supplies do have shorter lifespans, primarily because they're more complicated and thus have more failure vectors and because they have semiconductors exposed to upwards of 400V along with whatever mains transients make it through their input filters. it's doubly true for cost constrained designs where the capacitors are run near their ripple current and voltage ratings simultaneously.

>> No.1499772

>>1499719
ugliest machine ever seen on /diy/

>I even took the time to do cable management

it shows.

>> No.1499774
File: 59 KB, 1180x646, for amoeba.png [View same] [iqdb] [saucenao] [google]
1499774

>>1499757

>> No.1499796

>>1499772
jealousy doesn't suit you son

>> No.1499835

>>1499453
>>1499460
ah, got it
still mind your acute angles, where you have those thin traces coming into the power busses
personally I would stick to 0.3mm trace/space or greater if this is a hand-fabbed board, maybe even 0.5 if it's your first and/or you're using dodgy toner transfer paper
>forward annotation
I think that's how you're gonna have to do it. when I need to shuffle units, I usually rip up all the traces to the device in question and let the rat's nest view guide me, moving the highly-connected component around a bit if needed. don't be afraid to rip up traces entirely and redraw them, it's often easier than trying to modify what's there and very often produces a cleaner result
>1mm hand drill
now you see why you should have gone SMD

>>1499709
use pigment-based ink and feed your panel directly into the printer
*compatible printer and some assembly required

>>1499719
oi you got a loicense to grow that keyboard?
>3d printed from pla
shiggy diggy, anon. please use ABS at the very least
also
>kapton tape
if you're concerned about burning out the LED from overheat, you really should drill holes in the heat sink and screw the LED onto it with self-tapping sheet metal screws, instead of meme-tier attachment techniques like that which WILL fail

>> No.1499868
File: 99 KB, 2338x1653, unknown.png [View same] [iqdb] [saucenao] [google]
1499868

is a 0.8A inductor beefy enough in this circuit?
the output voltage on HV2 is 450V if that matters

>> No.1499898

>>1499868
i'm not familiar with that particular switcher and its weird high side current sense, but it says the current sense trigger is up to 350mV which corresponds to 1.4Apk. assuming a triangle wave (critical conduction, i'm not sure about loop stability for this shitty ic so i'd guess you want to keep it discontinuous) that's ~0.8Arms. so you ideally want a higher current inductor but you need to be sure that if you use the one you've got the core won't saturate at the peak current. have you considered using a chip with a built in gate driver, or better yet a built in 600V switch?

>> No.1499903
File: 41 KB, 937x507, 1534734503477.png [View same] [iqdb] [saucenao] [google]
1499903

>>1499868
did you do the math?

>>1499898
saturation current is a peak matter, not an rms matter

>> No.1500043

I have a small flat wheel with about a 1cm radius. On edge of this wheel is placed a neodymium magnet.
The wheel usually spins at around 1 RPS
can be a bit faster or slower
I need to calculate rotations of this wheel very accurately.
The wheel is placed in a plastic cylinder that is like 1.5cm across so there isn't much, space, i would have to use the small 5mm long reed switches.


Should i use a reed switch or a hall effect sensor?

>> No.1500051

>>1500043
Hall effect. Some boards you get will have a comparator on them and output a high/low binary signal which makes things easier, but you could just as easily calibrate your own with a hall-sensor, trimpot, and (schmitt trigger) comparator.

If you count the time between each pulse, invert it, and treat that as a moving frequency, you can use a weighted average with respect to time that has exponentially less weight on older samples than newer ones, and you can tune this exponential time constant to change the sensitivity/smoothness of your resulting average. But chances are you'll just use the arduino tachometer sample code anyhow.
t. never programmed an MCU

>> No.1500055

>>1500051
I will definitely be using an arduino.
It makes it so incredibly simple that i would be retarded for not using it just because ohm hispters hate on it solely due to its popularity.
I will be writing my own code though.
I was thinking of continuously monitoring the input, which will go up and down as the magnet distance changes and then simply add a +1 to a counter when the reading passes a preset limit and then pause the counter untill the rading underpasses a preset minimum limit.

But i just realized one huge problem.
I need to operate the thing on a battery for a long time and the hall effect requires constant power to work and at the same time it cannot be used as an interrupt to wake the arduino from sleep without needless complicated fuckery like op amps to turn the hall output into digital 0 or 1.
With a reed i can take a reading, put the arduino to sleep and let the next reading wake it up.

>> No.1500056
File: 25 KB, 485x443, trogloditlet.png [View same] [iqdb] [saucenao] [google]
1500056

>>1500055
>ohm hispters hate on it solely due to its popularity.

>> No.1500059

>>1500055
>ohm hispters hate on it solely due to its popularity
If that's what you legitimately believe, I feel sorry for you.

>> No.1500060

>>1500056
>>1500059
All i hear are memes and no actual arguments

>> No.1500061
File: 45 KB, 480x540, christ.jpg [View same] [iqdb] [saucenao] [google]
1500061

>>1500060
You use what you need to get the job done, I like using Nanos on my stuff, but more often than not I find myself reading the 800 page manual for ways to do think the interior designer friendly language doesn`t allow me to do. And Opamps are not fuckery, they simplify things, not make them more complicated. This kind of thinking will get you nowhere in any field related to EE.

>> No.1500065

>>1500060
It's fine if you want something simple to use for a simple solution. Good luck trying to understand why or how it's doing it though. This is primarily the problem with these plug and play solutions. Nothing is plug and play, and when you use a plug and play solution, you are at the mercy of their closed off system.
I personally have no gripes with it (especially since I tend to avoid coding), but it's naive to just dismiss those that don't like it as hipsters.

>> No.1500066

>>1500055
there are plenty which have open-drain outputs already so just connect to your MCU and go. you're still looking at 3-5mA current consumption, whereas the reed switch has zero standby current consumption
>using a counter
>very accurately
dude, no
you're gonna need to learn more about the chip and less about the lolduino to succeed at this project

>> No.1500067

>>1500065
How is this plug and play? All the componends are picked by me and code written from scratch by me as well.
>but arduino is a pre made component
do you make your own resistors? your own caps? your own ics? everything is premade.
and arduino has a ton of massive advantages. it is so popular that there are libraries and tutorials for everything if you hit a wall, or you can ask the large community and solve any problems within an hour. and it is also well documented in the field so you know exactly what kind of power needs for what you can expect etc..

>> No.1500072

I am going to go with the reeds since they have 0 consumption when the wheel is not spinning which is 70% of the time and i can put the ardunio to sleep most of the time when the wheel is spinning and just wake up for the counts. this will make the battery last forever and it will be super easy to mass produce too since i literally only need a single component to plug into the Arduino, which is the reed itself

>> No.1500084

I don't understand electricity.

I have an object emitting RF wrapped in aluminum foil. With a multimeter the voltage between it and ground is 5V. I made another thing with wires of unknown quality, put a AAA battery in the circuit, it reads 1.610V. I connect this object to ground. I connect the RF emitting object to it, and put the multimeter in series. The voltage is now 2mV.

Why would this be?

>> No.1500087

>>1500084
Also, the battery is no longer involved. That was just to test the wires.

>> No.1500100

>>1500084
So little actual information here,
Is the foil a transmitter or shield, how do you measure rf with multimeter, what is wire object, what polarity is battery connected, where in circuit is multimeter, what is rf out power and what drives it.
You don't measure voltage in series you measure voltage in parallel. A multimeter measuring just a battery the two are still in parallel.

>> No.1500109

>>1500100
The foil is an absorptive shield. I wanted to know if the ground prong actually had a path to ground, so I looked for nonzero voltage. I'm not trying to measure the RF, just verify that it isn't being rem-emitted or reflected. Wire object is a square sheet of aluminum foil with a wire taped to it. Multimeter has been between ground prong and shielded RF object, and between shielded RF object and wire object when it was connected to ground. Don't know RF out power, RF object is a computer plugged into the wall.

Measured in parallel the voltage was 0.

>> No.1500112

>>1500055
It's much harder to find an analog output hall sensor than a digital one.

Don't use an Arduino for this. You've gotta learn how to do stuff on your own eventually without a million inefficient libraries holding your hand.

>> No.1500114

>>1500072
>mass produce
>Arduino

Whyyyyy

>> No.1500135

>>1500114
it's super cheap, convenient, easy to use, and leaves a room for easy upgrades down the line
i won't actually be using arduino directly, but rather the wonderful esp32 since i has wifi and bluetooth which i need for remote readings

>> No.1500139

>>1500135
>cheap
Nowhere near as cheap as spinning your own boards with only the stuff you need
>convenient, easy to use
It doesn't matter how easy it is for you. Spending an extra week developing something you can make cheaper and smaller is always going to be worth it in a mass production case.

But you probably mean you're making ten of these for a science fair or something. Stop larping.

>> No.1500142

>>1500139
i'm not making millions, just about 100 which i will be using around my workplace
arduino is great because i can easily reprogram it or add other stuff to it
for some reason people don't understand that not everyone wants to spend 14 days by designing everything from scratch on component level just to save twenty bucks in the final result and on top of that give up the multiple advantages offered by a mass adopted product like arduino

>> No.1500151

>>1500142
You're... measuring rpms. Are you sure you're not overthinking this?

And what kind of information is this so that it's useful to collect 100 60RPM measurements

>> No.1500157

>>1500135
>esp32
>need it to run for a year
wew

>> No.1500159

>>1500151
it's an energy meter
rpm counting will only be one part of the whole thing, there will be other sensors attached too plus i want the ability to easily add additional hardware down the road if i wish so which the gpio pins and arduino ide allow me to do. i can literally just clip new sensors or modules on, clap on a cable to upload the new code from my android phone on the spot and am good to go
just thinking about the convenience and easy expandability it's offering me, is giving me a massive erection

>> No.1500160

>>1500157
if you are smart about when you turn on the wifi and how often you take measurements, while putting the module to sleep in between you can get years on a single battery easily

>> No.1500161

>>1500159
>I need 100 energy meters
????

>> No.1500167

>>1499289
You should get the Yihua 908D.

>> No.1500168

>>1500160
>esp32
>180mA in run mode
>yeeeears on one battery
hoo boy

>> No.1500174

>>1500161
My boss owns houses and rents apartments in them, we want to be able to take remote readings from various energy meters in the houses, and have them fed into a server in real time where i can whip up some code to analyze them and do interesting stuff with them
plus additional sensors such as ambient heat and other stuff

>> No.1500175

>>1500168
>>180mA in run mode
do you not know what a sleep mode is?

>> No.1500180

>>1500174
You're going to Jew people. Got it.

>> No.1500183

>>1500175
Not him but how much time it takes for an esp32 to register to the net, send some minuscule data packet and deregister?

>> No.1500184

>>1500183
couple of seconds, you simply don't send the date every time you measure but say every 2 hours, or even only twice a day if you want to really save up the juice

>> No.1500223
File: 3 KB, 164x228, Untitled.png [View same] [iqdb] [saucenao] [google]
1500223

Im looking for a plug that I can put in the wall and it redirects upwards. I got an extended outlet with usb slots on it, but it needs to be plugged into the top outlet and that is the one the light switch controls so our lamp is plugged into it, so I was thinking about getting a plug that just extends outward, and another that bends up at a 90 degree angle so I can plug it into the bottom while the top gets angled up behind it. Any idea if I can find this product or should I just make it?

>> No.1500323

what is the best (and cheap) way to switch between two sources of 220V?
Basically i have 220V inverter connected to a battery and regular 220V from a power company and an extension cable with some standard 220V devices.
Basically i want something like an electrical switch which i can "press" (for example with a relay or something) and it switches the power. But the two power sources musn't touch at any cost, so i can't just use a micro controller and two relay because the risk exists that something fucks up and both relay trigger at the same time blowing everything up

>> No.1500336

>>1499363
kek, bought the exact same model from a chineese store and last me 3 strong years

>> No.1500339

>>1500174
In this case there's no way you'd want to spend $3 on unknown Arduino clones when you could get brand-name SMT MCU ICs in bulk for less. I'd first check whether a reed switch at the appropriate distance will even trigger, since I bought a reed switch to actuate with a tiny magnet but the tiny magnet has to be less than 1mm away to trigger the damn thing because of how weak it is. It is for this reason that I suggested using a hall-sensor in the first place.

>>1500323
Run it on DC instead, it can probably handle it. Use a diode or two to ensure only the highest voltage (310VDC) gets to your load, but when that cuts off your boost-converted battery source (150-300VDC or so) will start passing power to your device instead. Literally zero danger, add a fuse if you're worried.

Most devices (ones without transformers or large motors) run switched-mode power supplies which simply rectify mains into 310VDC before switching them at multiple kHz, which makes them able to use a far smaller transformer than 50Hz could. For this reason, such devices often can simply run on both 110VAC or 220VAC with no switches needed. If what you plan on running with this includes a label like "100-240V~" (on both a USB wall-wart and my laptop charger) then you shouldn't have a problem. Sometimes these devices can be run at even lower DC voltages, down to 50V or 20V. Remember the √2 factor to convert between RMS (220VAC) and peak, because it's the peak that becomes the DC voltage when rectified.

I think the noise suppression circuitry within power supplies will work equally well in DC.

>> No.1500344
File: 79 KB, 906x526, FCO4G7UFHY0IAJ2.LARGE.jpg [View same] [iqdb] [saucenao] [google]
1500344

Just wondering if anyone knows how this pic gets 137VAC out of an Amveco 62045. Looking at the datasheet, it does have two outputs, but they both seem to be 22V https://www.alliedelec.com/m/d/aa12558a3e36db2444dd2f5d4cbc3d85.pdf

>> No.1500345
File: 49 KB, 760x435, FSAA2B6FHY0IAIF.LARGE.jpg [View same] [iqdb] [saucenao] [google]
1500345

>>1500344
That was the power supply for the amp in this pic.

Looks like the 320V and 305V lines were dropped to 305V and 295V. Would I be able to still use this PSU design for this audio design, or should I modify by adding something before/after the choke?

>> No.1500347

>>1500344
>>1500345
I'm guessing they picked the wrong part number. Any reason you're not going for a switching PSU (or 4) instead of a whopping great line-freq transformer? Otherwise I can't see the slight voltage difference being an issue.

>> No.1500349

>>1500344
the 137vac is the spare 115vac winding in series with one of the 22vac windings

>> No.1500352

>>1500339
i can't run "it" on dc it is multiple ac devices, i just need a way automatically switch between power sources while making it physically impossible for both to touch dicks

>> No.1500355

>>1500352
you should be able to use two DPDT relays. standard relays are break-before-make.

>> No.1500356
File: 9 KB, 332x174, file.png [View same] [iqdb] [saucenao] [google]
1500356

>>1500355
sorry one DPDT

>> No.1500380

>>1500352
that's still no reason not to just rectify mains and do it that way, inverters are a waste of money.

>> No.1500384
File: 261 KB, 930x550, HTB1qVWPINGYBuNjy0Fnq6x5lpXat[1].jpg [View same] [iqdb] [saucenao] [google]
1500384

>>1500355
>>1500356
nice this seems to be exactly what i need. i can just supply the trigger current from the mains so if the mains goes out the relay toggles to the inverter
>>1500380
i want the two power sources physically separated anyway for security. i am not going to fuck around with diodes on high mains voltages

>> No.1500388

>>1500347
I'm assuming it's something to do with avoiding switching noise and such? Found this design for a push pull tube amp (psu and audio stages) online and decided I'd give it a shot

>> No.1500389

>>1500384
I daresay a diode is more failsafe than a relay.

>> No.1500390

>>1500349
Thanks. I would never have thought to use one of the "inputs" as an output

>> No.1500392

>>1500389
failing short between the contacts is the failure mode that matters, and as far as i'm aware that's only possible in DT relays as a transient due to arcing.

>> No.1500395

>>1500389
>more failsafe than a relay.
the relay won't fail though the seller guarantees it
quote:
>The relay plays the role as a protector or a circuit switch. It can protect your electric equipments very well.

>> No.1500396

>>1500392
Which is why you put a fast-blow fuse in series with the diodes. Ensuring the circuit is lighting protected is probably a good idea, since a surge could kill the battery charge unit either way.

Also there are dedicated UPS + inverter combinations.

>> No.1500401

>>1500396
>Also there are dedicated UPS + inverter combinations.
those are intended only for the highest class of 1%ters

>> No.1500459

>>1500347
sag and other artifacts are audibly desirable in tube amplifiers, so a good stiff regulated switcher doesn't really help. also tubes, ilke JFETs, have very good high-frequency gain and, unlike JFETs, a lot more area with which to pick it up

>> No.1500478

>>1499028
>>1499047
>>1499120
Anyone have any design tips? I tried to lay out a guitar pedal PCB based on a circuit that I made in NI multisim once in Ultiboard, but I ended up having a lot of trouble with it and it just came out really sloppy.

>> No.1500528

What t12 tips should one have?
Something large for wire/connector soldering. T12-D4?
Universal 45deg cylinder. T12-BC2?
Needle for small components. T12-ILS?
Anything else?

>> No.1500543

>>1500339
He's using Arduino because he doesn't know the first thing about MCUs and doesn't want to admit that, or learn.

>> No.1500604

>>1500528
personally I would swap the BC2 and D4 with a D1.6 and a BC3. can't see any good reason for a 4mm tip other than large copper braid or maybe welding a copper box together
if you'll be doing much fine pitch (≤0.5mm) SOP/VQFP/DFN/QFN get a drag/hoof/wave/flow tip (same thing), T12-BCM2

>>1500543
especially in a project like this where the quantities start to become favorable for custom production and the off-the-shelf options are akin to trying to find the correct wrench with which to pound in the correct screw

>> No.1500626
File: 41 KB, 482x573, Screen Shot 2018-11-19 at 10.26.39 PM.png [View same] [iqdb] [saucenao] [google]
1500626

Rate my shitty shifter.
I'm working on a more optimized version right now.

>> No.1500667

Looking at 540 motors for an RC car, some of which are rated for as much as 90W. That would only happen with a load demanding half of the stall torque and running at half of the max speed, right? What current would be drawn in practice with, for example, a 16000 rpm motor and a 20:1 gear ratio? The ratio would mean less torque demanded of the motor and hence less current? Does current decrease as voltage is decreased in something approaching a linear fashion?

>> No.1500688 [DELETED] 

>>1500626

this is /diy/ dude. not /children-making-windmills-from-legos/. you incomprehensible mess of a drawing might be interesting to other child lego buildrs, but no one else. no engineer, fabricator or designer have ever spent 1 second of their lives trying to optimize a toy windmill.

>> No.1500689

>>1500626

this is /diy/ dude. not /children-making-windmills-from-legos/. you incomprehensible mess of a drawing might be interesting to other child lego builders, but no one else. no engineer, fabricator or designer have ever spent 1 second of their professional lives trying to optimize a toy windmill.

>> No.1500691

>>1500689
>windmill
what?

>> No.1500696

>>1500691
a building with sails or vanes that turn in the wind and generate power to grind corn into flour

>> No.1500697

>>1500667
Power rating is how much it can take before it'll overheat in what the manufacturer considers normal use, how much power it'll actually take depends on how you set up your ESC and batteries.

Got full specs?

>> No.1500699

>>1500696
no I mean how is >>1500626 a windmill

>> No.1500703

>>1500697
Don't really have specs. Just trying to figure out what class of motor I can use without going over ~2A. The intent is to skip the ESC and just give the brushed motor full voltage.

How about we look at the following. It says 3V working voltage with up to 1700mA draw which is 5.1W, but it is rated as 2W. Is that 1.7A the stall current? Would it be expected to draw considerably less under normal operation?

https://hobbyking.com/en_us/130-size-brushed-torque-tuned.html?___store=en_us

A further point of confusion for me - torque and current at the motor are determined by the load, but acceleration is determined by the torque delivered. How is that reconcilable?

>> No.1500704

>>1500697
>>1500699
On the last bit, is it simply following the linear torque-speed relationship, initially accelerating with the stall torque and steadily losing acceleration as it approaches the top speed where only enough torque remains to overcome rolling resistance? And so the current drawn is determined by whatever torque is delievered at the current speed of travel? Should the peak current at stall torque then be expected to briefly occur in actual operation?

>> No.1500708

>>1500704
Respond to wrong guy?

>> No.1500710 [DELETED] 

>>1500699
>no I mean how is >>1500626 a windmill

reference is to a toy system, unrelated to real-world needs, like a toy windmill, a toy car, a toy space-rocket, etc.

>> No.1500712

>>1500699
>no I mean how is >>1500626 a windmill

reference is to a toy system, unrelated to real-world needs, like a toy windmill, a toy car, a toy space-rocket, etc.
another common toy system we see all the time are those fucking binary adders they've been teaching schoolkids since granny was in school.

>> No.1500718

You know how chinks sell those 220V driverless leds which you can connect directly to ac?
Can i run those leds on 32V DC? Which is what they usually use if you buy them bare without any extra electronics on them.

>> No.1500729

>>1500718
you can run them off DC but to determine the minimum voltage you'd have to know the number of series leds in the string and ideally have the datasheet for the chink current sink ic. you can always just buy one and test it. undervoltage won't make it explode.

>> No.1500736
File: 308 KB, 584x700, small_dc_motor.png [View same] [iqdb] [saucenao] [google]
1500736

>>1500704
>briefly occur in actual operation?
Of course. The maximum current is determined by the supply voltage and the loop resistance, i.e. source resistance plus motor resistance. Pic shows a typical characteristic.

>> No.1500738

Fuck how there are so may inductors?
Is there any difference bwteen them if i want to use them for regular inductance purposes such as resisting current changes?

the store offers
Common Inductor
Power Inductor
Toroidal Inductor
SMD Inductor
Coil Inductor

>> No.1500742

>>1500738
you can use any of them as regular inductors except for the "common inductor", which is an annoying name but might refer to a common mode choke which is useless to you.

>> No.1500760
File: 424 KB, 640x640, 5PCS-LOT-UU9-8-UF9-8-50mH-50MH-Pitch-7-8mm-Common-Mode-Choke-Inductor-For.jpg_640x640.jpg.png [View same] [iqdb] [saucenao] [google]
1500760

>>1500742
common inductor is this, it looks like a transformer for some reason
what is it for?

>> No.1500764

>>1500760
you can`t go wrong with toroids m9

>> No.1500767
File: 27 KB, 616x427, 1410_emc_en28_3.gif [View same] [iqdb] [saucenao] [google]
1500767

>>1500760
Current to your device goes through one winding and returns through the other winding. If the coils are connected the right way, the inductance to your supply current is nearly zero, while inductance to common mode (equal and same sign in both wires) noise is high. It prevents your power cord or whatever acting as an antenna.
It is basically a special purpose transformer, used as an inductor.

>> No.1500781

>>1500703
stall current without an ESC limiting anything = battery voltage / (motor DC resistance + battery resistance + cable resistance). With these little motors going above max current for a bit is fine if you're not going to hold it stalled for minutes at a time.

Torque delivered is the load, acceleration will (ideally) be proportional to current.

>>1500704
Correct.

>> No.1500797

if i have multiple lead batteries and a solar panel with pwm charge controller do i connect the batteries to the controller in series or parallel?

>> No.1500799

>>1500704
Motors are limited by winding current. Voltage is limited by what the ESC can cope with (and make use of).

Torque is proportional to current. Current is limited to supply voltage minus back EMF divided by electrical resistance. Back EMF is proportional to motor speed. Acceleration is proportional to torque minus resistance (rolling resistance plus drag, but the former tends to be negligible).

At low speeds, the ESC is going to be intentionally limiting current to avoid burning out the motor windings. As speed increases, back EMF eats into the available voltage and limits the maximum current. Also, because the winding current is an AC waveform whose frequency is proportional to speed, winding inductance becomes an issue at higher speeds.

All in all, you use a supply voltage which would easily burn out the windings if applied to a stalled or slowly-rotating motor, so that the ESC will still be able to put a decent amount of current into the windings at higher speeds.

>> No.1500807

>>1500797
>pwm
connect the in a way that matches the voltage of the charger as close as possible. if your panels are 24v and you have two 12 bats then connect them in series to get 24% and therefore 99% efficiency on the pwm charger, that is even higher than mppt

>> No.1500814

>>1500807
that's not how MPPT works.

>> No.1500823

>>1500814
but it is how pwm works and mppt does not have 99% efficiency

>> No.1500824

Am i reading this correctly that to open this slut's pussy nice and wide i only need 1V at the gate?
That can't be right. I know that logical fets usually are like 5V or mabe 3V to saturate the gate but 1V seems too low, am i reading that shit wrong?
https://www.tme.eu/cz/Document/4e4b825d493d39061873a75efe1158b6/irl540n.pdf

>> No.1500825
File: 78 KB, 269x291, flux.png [View same] [iqdb] [saucenao] [google]
1500825

>>1500760
>common inductor
That must be chinkspeak. No one else calls a current compensated (common mode) choke a 'common inductor'.

>> No.1500826
File: 34 KB, 381x375, 1512240187745.png [View same] [iqdb] [saucenao] [google]
1500826

>>1500824
>Id=250µA
no, that's just the minimum price to stick it in, and only when she's hot (175°C)
Pic related is the graph you're probably interested in

>> No.1500827

>>1500824
>V DS = V GS , I D = 250μA
Yes, but only 0,25 mA of drain current is guaranteed at such low Vgs.

>> No.1500829

>>1500712
>>1500689
Who crawled up your ass?
He said it was a shitty design right off the bat and you still seem to need to feel like you're superior somehow.

>> No.1500836

>>1500826
so 3V gets me 10A? Nice thats perfect i only need 6A, so i can use my esp32 output pins directly to open it

>> No.1500839
File: 110 KB, 833x797, 1531937306577.png [View same] [iqdb] [saucenao] [google]
1500839

>>1500836
correct, but you will also be limited by the drain voltage. I'm a bit less sure about how this graph works

>> No.1500852

>>1500839
10 amps at 1 volt? that's fine

>> No.1500853

>>1500852
oh wait nevermind i read it wrong

>> No.1500858

>>1500852
yeah it's more like a 425mV drop across at 6A with Vgs=3V

>> No.1500862

>>1500858
does that mean it will get 6*0.425 W hot?

>> No.1500867

>>1500862
ja. you will need a heat sink but a pretty small one will do. I bet you could get away with one of those raspberry pi heat sinks just adhered to the back

>> No.1500881

>>1500867
ok, thanks hanz

>> No.1500914
File: 51 KB, 720x960, 1542662038173[1].jpg [View same] [iqdb] [saucenao] [google]
1500914

Would this actually work?
If not, how much dirt you actually need?

>> No.1500917
File: 83 KB, 600x600, 0C686FA8-E4CE-4B0C-9B2D-324ECF9EDB54.jpg [View same] [iqdb] [saucenao] [google]
1500917

I’m working on this project I found in this link. https://jonasbers.com/chav/
It’s supposed to be a video synthesizer made out of a VGA tester board. My problem is that the board doesn’t seem to be getting power when I plug in the AC adapter. The little red light won’t come on and nothing comes up on the monitor. What might be giving me this problem? Pretty new to this so go easy on me.

>> No.1500921

>>1500914

Yes...assuming your dirtball is at least a few quadrillion tons in mass.

>> No.1500923

>>1500921
so does that mean i can take an applicance and stick the neutral wire coming out of it into the ground outside and it will work?

>> No.1500926

>>1500781
>>1500799
I've been having a hell of a time getting useful information googling, so that's very helpful, thanks.

How is current limited by an ESC? It looks like only higher end ones actually have a current limiting circuit and cheaper ones will simply burn out and save the motor/battery. What will happen if the motor tries to draw more current than a nimh battery can supply without an esc? I was looking at datasheets for AA nimh and it looks like they can operate normally at as much as 6A unsustained (3C), but a 540 motor would possibly get as high as 20-40A (I can't find data for smaller motors). Would the batteries rapidly die or could they handle such spikes and still manage to run such a motor for several minutes?

>> No.1500932

>>1500923
depending on where you live and what the setup is normally neutral and earth are bonded somewhere at the incomer of an installation.
dry dirt probably has a much higher resistance than copper wire however a properly commissioned earth rod should have sufficiently low resistance path to allow overcurrent protection to operate.

some people living on remote e.g. farms only get a phase wire and return (neutral) is through the earth.

>> No.1500943

>>1500926
>What will happen if the motor tries to draw more current
so something can't really draw current right? we apply voltage, the device has an impedance, the current is voltage/impedance. the simplest way to limit current is to limit voltage and let the current limit itself.
batteries have an internal resistance too which you can use to work out what happens when the impedance is too low or voltage is too high, the battery internal resistance acts with the load as a voltage divider so some voltage is dropped across the battery which is realised as the voltage output of the battery will drop as it can't deliver enough current.

>> No.1500959

>>1500943
Problem is the internal resistance varies considerably, increasing with age and other crap. I guess to simplify things, the real concern is heat generation. The battery will go up to some max current, limited by its own resistance, and will rapidly overheat if allowed to stay there, but otherwise will operate (with much shorter life). Correct? In short, I can slap whatever motor on whatever battery as long as I pay attention to things not overheating?

>> No.1500961

>>1500932
And when it rains all lights are brighter.

>> No.1500964

I can't get my shitty reed switch to trigger properly, what is going on? It is not damaged.
I stick it into two female dupont breadboard wires. Could it be that because the dupont terminals have magnetic (but not magnetized) metal in them it somehow throws of the reed switch?

>> No.1500966

>>1500829
>need to feel like you're superior somehow

if you had some reading comprehension skills, you'd have understood the VERY SIMPLE point i was making is that this is a board for builders and fixers, so someone discussing theoretical homework-like shit would be better served elsewhere.

>>1500926
>Would the batteries rapidly die or could they handle such spikes

batteries have an internal resistance, which isnt a real resistance but rather a measure of how quickly it can turn chemical energy into electricity. AA nimh's cant handle 20A spikes coz the internal resistance is too big. and if you're pushing them hard to get 6A out, you can be sure the output voltage is nowhere near 1.2V, but close to 0V, so not much actual power is being used.

>>1500923
>and it will work?

work in what sense? by pulling current through the ground? or for ground protection? the former is actually used in some remote places where they deliver electricity on a single wire. apparently you can get shocked lying on the ground, or swimming, coz there's enough potential voltage between two spots like 1 meter apart.

>> No.1500968

>>1500964

the very cheapest dupont wires use steel instead of being pure copper, so it could be they're stealing and storing some of the magnetism.

>> No.1500970

>>1500968
I see. Even though my duponts are made from the highest quality chinesium materials, the could have some steel in them since they stick to magnets like glue.

>> No.1500973
File: 1.76 MB, 640x480, 8nrXK25.jpg.gif [View same] [iqdb] [saucenao] [google]
1500973

>>1500932
>return (neutral) is through the earth

If they fed phase into ground and ran a neutral air-wire, would it work?

>> No.1500976

>>1500973
No. Electricity flows from up hill to down hill.
You would be trying to make the water go up hill.

>> No.1500977

>>1500976
>>1500973
Although that would be a conventional water molecule flow, the actual water in real world moves from down hill to up hill.

>> No.1500988
File: 253 KB, 1641x1212, shit.jpg [View same] [iqdb] [saucenao] [google]
1500988

I am going to require some adult supervision.

I just cannot make this piece of shit work.
This is a water meter.
As the water flows, the magnet, marked in the picture, spins.
The magnet is actually intended for use with sensors, so they can measure the water consumption, so it's not like i'm trying to use something that wasn't designed for this kind of use.
I basically want to count the turns of the magnet with a reed switch in the photo.
But for the fucking life of me, i cannot find a position to put that damn reed switch in, in which it consistently triggers as the magnet keeps turning.
Sometimes i get it to trigger, but mostly it doesn't at all.
What am i missing here?

>> No.1500998

>>1500055
couldn't you do the same thing with an MSP430fr6989

>> No.1501000

>>1500998
See >>1500543

>> No.1501001

>>1500977
lol

>>1500973
or drive the stake so far it hits the aquifer, so the electricity can travel back to the plant via the ground water.

>> No.1501002

>>1501001
wouldn't the fish get shocked

>> No.1501009

>>1500988
Possibly magnet is too weak to comutate a reed switch reliably. You can try 2 things: replace it with Hall effect sensor, or cut aray plastic ring around magnet`s housing to bring reed switch clouser to it

>> No.1501011

>>1501009
can't cut the plastic, there is a cap seal that goes there which will get sealed later
i can't use a hall either since it requires constant energy to run unlike a reed which requires none and the meter will not be spinning 80% of the time and my shit will be battery powered.

>> No.1501017

>>1501011
i have ordered some 5mm chinesium reeds to see if i have any more luck, since these are 1.4cm so they are huge and can't fit for shit

>> No.1501019

>>1501011
There are hall switches which draw less than microamp. I doubt your other shit's power consumption is low enough for that to matter.

>> No.1501020

>>1501019
such as?
they can't be microscopic though, i need to solder them by hand to a piece of wire

>> No.1501026

>>1501020
DRV5032 comes in SOT-23, which shouldn't be too difficult to solder even for ham-fisted guys.
If you insist on through hole components, there's for example A3211, but its current consumption is around 6uA.

>> No.1501029

>>1501026
Looks nice. If i can't get the reeds to work out in the end, i will try the halls. Hopefully they will be more sensitive, but no too much since i don't want them triggered all the time, when they are within the confines of that plastic cylinder.

>> No.1501032
File: 33 KB, 732x398, Beginnings of a CPU.png [View same] [iqdb] [saucenao] [google]
1501032

>>1500966
>This is a board for builders and fixers
And I'm building a 8-bit CPU, so fuck off.
The end goal is to build it from 74AS series logic chips.
I've completed the specification for the instruction set, and now I'm working on implementing it.
Once I finish implementing it, I'll order the ICs and start assembling it.
The ALU's shifter just happened to be what I'm working on at the moment.

Now, are you going to keep tilting at windmills every time I post an update, or will you cool the autism?

>> No.1501038

>>1501002
electricity follows the path of least resistance. Fish are friends, not food.

>> No.1501040

>>1501029
What do you know about the magnetic orientation of the rotating magnet?

>> No.1501044

>>1501038
>electricity follows the path of least resistance
You need to work on your anthropomorphic memes. Current follows _all_ paths according to conductance.

>> No.1501049

>>1501040
Nothing. Only that it is a neodymium magnet. The thin sides seem to trigger the sensor the best

>> No.1501051

>>1501032
>The end goal is to build it from 74AS series logic chips.

how'd you pick that?

why not transistors and diodes?

why not mechanical relays?

why not wooden gears?

what, are you gay or something?

>> No.1501061

>>1501051
I thought about doing DTL, but it would be too expensive and time consuming for me to make the processor as complex as I've designed.
Relays are also neat, but even more expensive.

I chose 74AS because they're faster than 74HC, and not a whole lot more expensive.
I might also port it to an FPGA and see how much the FPGA outperforms the discrete implementation.

>> No.1501129

>>1501044
and you need to work on your shitpost recognition

>>1501061
>port to an FPGA
>he doesn't know about HDLs
stocks in popcorn manufacturers rose 1% that day

>> No.1501141

>>1500823
MPPT gets you more power from the panels, so while you might be losing a few percent more in the conversion, you're still getting more to the battery.

>> No.1501147

>>1500807
that's not how battery charging works
or "12V" batteries, either
and it's sure as fuck not how overcharging protection works

>> No.1501149

>>1500959
Correct. With that motor it's not gonna be a problem if you're using something like 3xAA NiMH, if using quite a bit smaller cells you might have to check for

>> No.1501204
File: 70 KB, 484x454, Screen Shot 2018-11-20 at 10.51.11 PM.png [View same] [iqdb] [saucenao] [google]
1501204

>>1501129
>Implying I don't know about HDL
Do you honestly think I'm that retarded?

I haven't written any HDL for this project yet because it honestly isn't that useful for actually implementing the design in discrete logic.
Ergo, I would need to write the HDL to "port" it to a FPGA.

Here's some VHDL I just wrote, if it makes you happy.
And yes, I realize that sll uses a janky non-standard definition. I don't care because I'm not actually going to use this code.

>> No.1501242

do inductors work like magic?
Say you have a dc cicruit.
A source --- led ---- ground
If the soruce's voltage surges for a bit the led is a toast
So if i do this
A source --- inductor --- led ---- ground
Will the circuit work exactly the same as before except now its magically protected against short voltage surges?

>> No.1501244

Is infrared safe for human eyes?
I would like to add a 100W infrared LED as an invisible light for my night camera, but I don't want to go blind by looking into it.

>> No.1501248

>>1501242
>magically protected against short voltage surges?

yeah, very short surges will be resisted as the coil uses the extra energy to build up its magnetism. but there's better ways that'll handle short, medium and long-term surges. like zeners, voltage regulators, metal oxide varistors, etc.

>> No.1501258

>>1501244
High power IR can be hazardous, more so than visible light of same power since you're not going to get the iris shrinking. Osram has a good application note on figuring out the limits https://dammedia.osram.info/media/resource/hires/osram-dam-2496541/Eye%20Safety%20of%20IREDs%20used%20in%20Lamp%20Applications.pdf

>> No.1501269

>>1501248
what is considered very short?

>> No.1501281

>>1501269
A coil does not remove the energy of the surge and will not protect your LED. It can only delay the overload because the temporalily stored energy has to go somewhere.

>> No.1501289

>>1501269
Depends on lots of things, a coil is a low pass filter, doesn't absorb spikes it just smoothes them out. You can look up equations for filters depending on the topology.

>> No.1501340

So measuring battery capacity is easy, just read the voltage with say arduino and compare it to a corresponding charge % with a table.
But what if the arduino is powered from that very same battery?

>> No.1501343

>>1501340
then you're measuring it under load

and it's always relative to some reference voltage constant

>> No.1501344

>>1501343
>relative to some reference voltage constant
I don't understand

>> No.1501364

>>1501340
Getting a rough indication is easy, but if you want any accuracy, particularly under varying load, it's not that easy anymore.
Tarduino has its own regulator which can be used as the ADC's reference. If the battery voltage is high enough for the regulator, you can compare the battery voltage to the regulated voltage. If that's not possible, you can use the processor's internal 1.1V reference voltage. Either way, you have to scale the battery voltage accordingly before feeding it to processor's analog input.

>> No.1501372

>>1501344
The ADC (Analog to Digital Converter) in the Arduino measures 0 to 1024, where the scale is defined as 0 = 0v and 1024 = the reference voltage.

>> No.1501373

>>1501364
what discrepancy can i expect under load? i do not need it to be surgical but it has to be at least within 8% of precision

>> No.1501378

>>1501372
Oh i see so 2.2V would be 2048 if you use 1.1v reference

>> No.1501380

>>1501378
>2048

lol. at some point it would behoove you to actually look at an avr spec.

when he said 0..1023, that's what the range is. it's a 10 bit ADC.

>> No.1501381

>>1501373
That depends on the battery's internal resistance, and the internal resistance tends to increase as the energy gets used.

Here's how to measure the internal resistance:
Place a resistor across the battery and measure the current flowing, then measure the voltage across the resistor.
Now subtract the voltage you measured from the nominal/max voltage (1.5v for a alkaline, 4.2 for Lithium ion).
You can use the voltage difference and current you measured to calculate the internal resistance.

>> No.1501447
File: 65 KB, 921x549, battery-discharge.png [View same] [iqdb] [saucenao] [google]
1501447

>>1501373
It depends on your battery, its age and use and the temperature.
Pic related is a pretty typical voltage vs. charge curve for a lithium battery. Note how a significant part of the low current discharge curve is very flat, so that even a small error in voltage measurement easily produces a large error in the predicted capacity. As can be seen, the shape of the curve is also dependent on the load and the internal resistance isn't constant either. In short, 8% error is not easy to achieve. If you want better results, you should measure the actual charge. There are plenty of "fuel gauge" ICs for that purpose, for example TI's BQ35100.

That said, voltage measurement is much easier with alkaline batteries.

>> No.1501453

>>1501447
And what magic do those ics use then? Surely they have to use the voltage measure as well.
>>1501381
Well that won't do me much good since i will be using various mix of batteries

i just found a genius and incredibly simple solution.
connect a cap, put arduino to sleep, let battery charge cap, wake arduino, read voltage across cap, BOOM perfect precision charge measure

>> No.1501463

I haven't done any electronic since highschool, and I'd like to practice a bit again. I was thinking of making a pair of radio transmitter and receiver from scratch. Something very simple to begin.
Am I correct to assume that this will require an oscilloscope to make sure that my circuits are working properly ?
Is there any cheap but reliable brand/model that you would recommend to a beginner ?

>> No.1501488

>>1501453
>Surely they have to use the voltage measure as well.
They typically integrate charge. Fancier ones, like the chip I mentioned, can also measure current, voltage and temperature.

>> No.1501524

>>1501463
>oscilloscope
if you want cheap, and don't mind it being a bit slow, you can just get a voltmeter

>> No.1501540

>>1501447
>BQ35100
That is for non-rechargeable (primary) lithium batteries.

>> No.1501555

holy shit i now understand how voltage dividers work, it just clicked, fucking finally!

>> No.1501556

>>1501555
you did it!

>> No.1501561
File: 91 KB, 800x558, how do they work.jpg [View same] [iqdb] [saucenao] [google]
1501561

>>1501242
>do inductors work like magic?
inductors are magnetic
magnets are magic
ergo: inductors are magic

>> No.1501563

>>1501524
That wouldn't allow me to verify the frequency of a signal, would it ?
Is there anything that can do that beside an oscilloscope ?

>> No.1501566

>>1501563
decent ($40+) meters can measure frequency. neither a meter nor a hobbyist scope will be able to measure RF signals though.

>> No.1501570

>>1501566
>neither a meter nor a hobbyist scope will be able to measure RF signals though.
Are those frequencies too high ?

I guess I could start with fairly low frequency montages to make sure that I understand what's happening with the help of whichever measuring instrument, and then move up to higher frequencies "blind".

>> No.1501573
File: 79 KB, 800x800, HTB1NQTxXjnuK1RkSmFPq6AuzFXaf[1].jpg [View same] [iqdb] [saucenao] [google]
1501573

fuck, this chinesium power sauce is really tiny, only like 2cm across.. s..should i really feed 230V into it?.. i am so going to die..
i hope it won't get super hot and melt when i try to pull 1A from it at 5V dc output

>> No.1501584

>>1501573
Connect a 5Ω/5W resistor and see what happens.

>> No.1501597

>>1501573
>only like 2cm across.. s..should i really feed 230V into it?..

not relevant in any way. If it were as big as a barn it could still be dangerous if designed or manufactured improperly.

>> No.1501603

>>1501573
If you are such a pussy, connect a 100W tungsten lightbulb in series with power supply in question. If it is ok - lightbulb will flash slightly when you plug it in, and then stay dark or dimly lit. If PS is faulty - lightbulb will light up and prevent a ka-boom

>> No.1501604

>>1501573
if you're worried, you can do a bit of reverse engineering and learn something along the way. the safety factors are:
>clearance between isolated groundplanes
you can measure this yourself with calipers and compare to creepage.com
>insulation between primary and secondary transformer windings
probably just the wire enamel and a layer of kapton. you get what you pay for but i wouldn't trust anything important to it because heat from a sustained short or control fault can result in breakdown.
>presence and quality of any filter caps bridging the isolation gap
in this case there's probably no such caps, which is better than having bad caps there.

>> No.1501605

>>1501603
*in series with main's cirquit of

>> No.1501615

>>1501566
You serious?
I got a 200mhz scope for like $400.
That's good for everything south of the 5m band

>> No.1501622
File: 11 KB, 300x225, Digital-Multimeter.jpg [View same] [iqdb] [saucenao] [google]
1501622

>>1501555
Now you need some practice. What voltage will the meter display?

>> No.1501629

>>1501622

now that's just mean. he spent this semester learning about voltage dividers, and will study parallel resistors next semester.

>> No.1501635

>>1501615
>I got a 200mhz scope for like $400.
good for you, how many of your friends got that deal?

>> No.1501640
File: 306 KB, 2048x1536, IMG_20181121_230221.jpg [View same] [iqdb] [saucenao] [google]
1501640

>>1501597
>>1501603
>>1501604
well... it's doing something...the red light is on, not sure what though, i am too scared to move too close to it with a multimeter because i just know the second i get close to it, that very likely undervoltaged 230v cap is going to explode in my face, spewing boiling electrolyte all over me, turning me into a hideous twisted tormented monster for the rest of my life

>> No.1501642

>>1501640
>i am too scared to move too close to it with a multimeter

excellent. forget electronics and go back to whatever you did before.

>> No.1501644

>>1501642
No. I love working with low voltages because i can touch them without fear but i am fucking terrified of high voltage AC
After i got zapped by it once i got a phobia ever since.

>> No.1501646

>>1501644
>No. I love working with low voltages

then stop fucking with 230 volt devices.

which part of this confuses you?

seriously. start your own thread "electronics for dummies" and get the hell out of this one

>> No.1501647

>>1501635
your mother got the deal, but only because she gave me a deal.

>> No.1501651

>>1501646
>then stop fucking with 230 volt devices.
do you not understand where low voltage dc comes from in your home?
try asking your dad about it, or his sister as in your mom

>> No.1501654
File: 54 KB, 1092x752, PPS cable diagram with arrow.png [View same] [iqdb] [saucenao] [google]
1501654

Can I safely run only the RX/TX and ground lines through this UPD4721 chip while ignoring the rest of the circuit? Or is the DCin and caps necessary for creating the correct ground state?
I am a layman so I apologize in advance.
For context, this is a radio programming cable from a long dead company.

>> No.1501655

>>1501651
>do you not understand where low voltage dc comes from in your home?

THIS BOARD NEEDS THREAD SPECIFIC IDS SO THAT I CAN TELL WHICH RETARD I AM REPLYING TO.

of course I know where low voltage dc comes from

are you actually defending that moron? why?

>> No.1501684

I have to connect stranded 6 AWG to stranded 8 AWG; is a reducing butt connector the best way to do this or should I look at another solution?

>> No.1501687

>>1501654
It is safe but it will not work.

>> No.1501690

>>1501687
Thanks.
Is it because the DCin drives the functionality of the not gates?

>> No.1501691

>>1501684

who writes this comedy

>> No.1501694

>>1501691
My dad, who doesn't like to listen to the advice of others.

>> No.1501707

>>1498861
Close. It's called using blocking diodes. This is how we wire multiple redundant supplies in parallel in industrial cabinets. It's stops a supply from becoming a load and starting a fire.

>> No.1501723

I'm looking at the book: Learning the Art of Electronics

And the parts list online comes out to like $500. Does anyone know if that's reasonable? It kinda breaks the bank a bit.

>> No.1501724

>>1501684
Terminal strip or wire nut?

>> No.1501743

>>1501573
>i hope it won't get super hot and melt when i try to pull 1A from it
>>1501640
>i am too scared to move too close

Why did you buy it?
You should go into woodworking. -- OH WAIT, that has sharp blades...
Just take a nap.
>but people die in their sleep!

>> No.1501751

>>1501724
It's on a boat and kind of a tight fit so having it sealed up is important. I'm helping my dad outfit his boat with a new marine SSB and even though the control head and the transceiver can be up to 15' away he still insisted on mounting them together at the chart table instead of using the mounting bracket to put the transceiver in a location with better ventilation and closer to the batteries. Consequentially, the length of 8 AWG that the radio shipped with isn't long enough and he bought some 6 AWG to make up the difference.

>> No.1501787

Has anyone tried this?
https://youtu.be/Z4zO2gB70ps

>> No.1501818 [DELETED] 

>>1501690

the 78L05 is needed so you can power it from a multitude of voltages, from 6 to 12V. but if you have a 5V supply, like a phone charger, you can eliminate the 78L05 and feed pin 3 directly from that. the multiple capacitors are all needed to generate the high voltages (usually +12V and -12V) that the RS-232 lines need.

>> No.1501822

>>1501690

the 78L05 is needed so you can power it from a multitude of voltages, from 6 to 12V. but if you have a 5V supply, like a phone charger, you can eliminate the 78L05 and feed pin 3 directly from that. the multiple capacitors are all needed to generate the high voltages (usually +12V and -12V) that the RS-232 lines need.

if you cant find this old chip, you can use a MAX232 instead. it's wired differently but does the same job.

>> No.1501836

>>1501822
I actually did manage to get a bunch of them from a radio shop in the Czech Republic but I'm still going to file that alternative away for later use. Thanks for your help, it's appreciated.

>> No.1501858
File: 71 KB, 800x442, pioneer_sa-6800_stereo_integrated_amplifier.jpg [View same] [iqdb] [saucenao] [google]
1501858

Picked up a Pioneer SA-6800 for cheap, but it seems one of the channels is weak. I played a 523.25Hz (C5) sine wave out of it, and adjusted the volume until the louder speaker measured 100mV across it. The quieter one measured 35mV.

I then swapped the channels, and did the same measurement. While the louder speaker was at 100mV, the quieter speaker measured 39mV. Since the issue stayed with the amp channel instead of the speaker, I realized it must be the amp.

Looking at the schematic on page 16 of the service manual here https://www.hifiengine.com/manual_library/pioneer/sa-6800.shtml I think my next step should be to measure the base-emitter voltages of the four output transistors (two 2SB705A and two 2SD745A). They should all be around the same voltage, right?

And if one's significantly off, I'll compare its collector voltage to the others, to see if it's even getting the right signal. Am I on the right track?

>> No.1501859

>>1501858
thou shalt measure voltages is definitely a good place to start.

>> No.1501861

>>1501540
kek
Maxim's lot for rechargeable Li+
https://www.maximintegrated.com/en/products/power/battery-management/battery-fuel-gauges.html

>>1501563
a radio
an SDR
the FCC dropping by your house

>>1501644
you should really have alligator clips for your probes so that you can stay a safe distance away from the process
>tfw the real fun begins when anon disconnects the device from the mains and thinks it's safe and inert

>>1501654
yes

>>1501723
try pricing them out in 0805 SMD, they might be a lot cheaper

>>1501858
with no input applied, measure dc voltages on all the terminals of the transistors, relative to ground, then *plot them on a copy of the schematic*, both the good channel and the bad channel
if no "wrong" voltage is obvious, check capacitor voltages, then try to find good resistors to measure current

>> No.1501864

>>1501858
Also, would it be worth replacing all of the caps with a $37 (shipped) kit like this? https://www.audio-high-store.com/product/pioneer-sa-6800-upgrade-kit-audio-capacitors/

I know I can get the caps individually from Mouser, but I'd prefer to just save myself the trouble of generating a list of capacitance/voltage/material/lead spacing/etc.

>>1501859
K, I'll see what I can do after the holiday

>> No.1501868

>>1501864
>would it be worth
that's a value judgment only you can make

>> No.1501870

I'm looking at tiny SMT h-bridge ICs with crazy output power specs. This is a good example:
https://www.digikey.com/product-detail/en/texas-instruments/DRV8872DDAR/296-42661-2-ND/5428829

45V, 3.6A. How can that be? Does it have extremely low Rds, so little power dissipation in the device? Or is this spec more like "it'll handle this as long as you keep the junction temperatures low" and then it's on the user to heatsink them?

>> No.1501872

>>1501868
I also lack the proper knowledge to identify which caps would need to be swapped and which can stay. I think I'll get it, and study which caps they chose to swap and which they omitted.

>> No.1501876

>>1501870
The Rds isn't that low, but it can be run pretty hot (limit at 175C die) and has a thermal pad underneath you're expected to solder to the board with big copper areas and vias through to the other side as the heatsink, getting enough copper to sustain 3.6A continuous may not be practical.

>> No.1501878

>>1501876
Do you think I'd do significantly better by using discrete through-hole FETs? Are there ICs that'll do the negation, charge pumping, etc., to interface a microcontroller to the FETs?

>> No.1501885

>>1501872
power supply caps go quickest due to the ripple current heating. if you can hear excessive hum and all the bias points look correct, replace these
as for the other lytics, re-cap if they look swollen, damaged, or leaky, or if you've reason to believe they're not functioning properly (i.e. tacking a near-equivalent from your parts stock across it drastically improves performance)
see also https://en.wikipedia.org/wiki/Capacitor_plague

>>1501870
>it'll handle this
peak, but suggested 2A continuous (section 8.2.2.2)
>it's on the user to heatsink them?
yes, but thus with all SMD, and especially those with exposed pads. a suitable ground plane and maybe a stick-on bit of finned aluminum might be enough to disspate 2A * (565mV * 2) = 2.25W
please to read section 10 of the datasheet

>>1501878
those are filed under "gate driver", e.g. FAN73832

>> No.1501888

>>1501878
You can get quite a bit lower Rds with discrete FETs and a driver chip, the main advantage of the H bridge in a chip ICs is being tiny.

>> No.1501890

>>1501888
Someone needs to hurry up and invent SMD devices that dissipate more heat than THT already.

>>1501885
Thanks

>> No.1501891

>>1501890
That SMD already can dissipate much more than a same size THT part.

>> No.1501893

>>1501891
Then we just need SMT parts that dissipate as much heat as bigger SMT parts. Shouldn't be hard

>> No.1501895

>>1501891
Wait, so SMT or THT discrete FETs? I'm thinking two layer board, good amount of room for copper pours, no forced airflow.

>> No.1501897

>>1501895
If you've got space to go vertical THT can be easier.

>> No.1501900

>>1501897
Easier how? My priority is power

>> No.1501903
File: 110 KB, 1280x960, 1522312157974.jpg [View same] [iqdb] [saucenao] [google]
1501903

>>1501900
first decide how much power then select components to meet the need
otherwise your priority leads you to massive IGBTs with screw terminals and hundred-dollar price tags

>> No.1501915
File: 3.51 MB, 7752x3453, hfe_pioneer_sa-608_6800_service-016.jpg [View same] [iqdb] [saucenao] [google]
1501915

>>1501885
As far as the capacitor plague, are you implying the kit may come with bad caps?

And here's the schematic. I'll check out TP1, TP2, TP3, and TP4. Looks like there are some voltages are already marked, that's a big help.

>> No.1501921

>>1501915
nah, actually just observing that caps do have a finite lifetime. how old is this thing anyway?
>well-labeled schematic with nominal voltages everywhere
this should be a piece of cake. last receiver I did as a teen, I didn't even have that much

>> No.1501940

damn, the cheap invertor i ordered that was supposed to do 3k W shuts down at even 2k W according to reviews.
Well luckily i counted on that and ordered a 3k version when i only need to draw about 600w. hopefully i managed to underestimate this quality chinesium product low enough to be actually within its limits

>> No.1501975

>>1501684
>>1501751
A butt connector sealed inside heat-shrink tube would be my go-to solution in your situation.

>> No.1501980
File: 84 KB, 1000x1000, 1523482598908.jpg [View same] [iqdb] [saucenao] [google]
1501980

I have these cheap piece of shit Chinese dimmers on my LED strips, and I love having a dimmer, but these are flickery and scale the light really weird. They're just big potentiometers, right? How do I determine what size pot I need for my LEDs so I can just order good ones on Arrow? I want to replace having 4x of these mounted on my wall with a nice little custom box.

>> No.1501990

>>1501980
>They're just big potentiometers, right?
Not exactly, more like PWMs unless they're real el cheapo ones. The fact that the light flickers leads me to believe this is the case with yours - if it just limited current it shouldn't do that.

>> No.1501991

>>1501990
And to clarify, I mean that the dimmer in question most likely isn't just a potentiometer.

>> No.1502009

>>1501991
>>1501990
>real el cheapo
I think they were about $2/ea on AliExpress. I'm really wary of PWM. We have a fixture that's PWM, on a proper dimmer not just 60hz, and any movement has a noticeable strobe effect in that room. I've also heard that it can cause interference, and I don't know enough about that to know if I should care.

Maybe I'm coming at it the wrong way, and trying to fix a specific thing instead of looking at the whole picture. Is there a better way to dim LED? If you wanted it done up proper, how'd you go about it?

>> No.1502027
File: 4 KB, 225x225, pot.jpg [View same] [iqdb] [saucenao] [google]
1502027

>>1502009
No flicker, no EMI or RFI. Supply with constant voltage, not constant current.

>> No.1502041
File: 616 KB, 2048x1536, IMG_20181122_143250.jpg [View same] [iqdb] [saucenao] [google]
1502041

I will be testing my photon baby today for the first time, wish me luck . Maybe i will accidentally lasik myself and won't need glasses anymore, that would be cool

>> No.1502043

>>1501751
self-amalgamating tape is lovely

>>1502009
>proper dimmer
only 100-120Hz at most, I can imagine how that would be strobey
>how would you
PWM dimming with a frequency of ~250Hz or more. electrical infetterence is not much of a factor at 12V, only where there's a switching constant-current converter involved where significant harmonics can land in the SW/MW bands

>> No.1502074
File: 91 KB, 1000x689, HTB1.HWLLXXXXXaXapXXq6xXFXXXt[1].jpg [View same] [iqdb] [saucenao] [google]
1502074

>>1502041
Correct me if i'm retarded, but will shorting the till pins safely achieve 0V permanently?

>> No.1502075

>>1502074
Basically the problem is that if i supply 5V on TIL then the laser is off, this seems to be correct as per the pic
but if i disconnect the + wire from til OR both wires from till the laser is still off and only smetimes flickers on for a short moment

>> No.1502077

>>1502074
correct
it's bad form to leave logic inputs floating

>> No.1502080

>>1502077
I don't get it. I tried a 100k resistor between the pins and nothing, then i tried to short them directly and also nothing

>> No.1502083

>>1502080
>short them directly and also nothing
that sounds broken

>> No.1502086

>>1502083
the fan on the laser is spinning and a green led on the control board is on
also when i connected the laser for the first time it turned on and i was able to hold it and cut paper for like 60 seconds, before i cut the power
also just looking at the control board it looks fine, nothing burned or misplaced

>> No.1502089

>>1502086
it is something with the driver board, i tried disconnecting the laser cable and measure voltage across it's power supply pins on the control board and it's 0

>> No.1502092

>>1502089
i wonder what the potentiometer does.. i am kind of scared to fuck with it in case this is something small and by fucking with it i will break it for real

>> No.1502101

What am I doing wrong?
I've got some LEDs with a 3.3V drop and rated for 150mA.
I'm trying to run them from a 24V supply.
So with 6 in series, there should be a total drop of 19.8V, leaving 4.2V to be dropped across a resistor.
I'd a 33 ohm 1 watt resistor handy so I thought 4.2/33= 127mA will be fine for the LEDs.
However, when I applied my 24V, thw LEDs burnt out one by one within a minute.
It shouldn't matter if the resistor is on the positive or negative supply should it?

>> No.1502102

>>1502101
Now that I think of it, the other LEDs stayed on after one blew even though they're in series.
It's not standard for a LED to fail closed is it?

>> No.1502114

>>1502092
welp i give up.. i tried everything and it's not working.
and i had it home too long without testing it so now i am past the period when i can return it.
I am going to try to buy a new driver board
This one seems a right fit:
https://www.aliexpress.com/item/12V-TTL-1W-2W-3W-445nm-450nm-Laser-Diode-LD-Driver-Power-Supply-Stage-Light-free/32627087870.html
It only costs 9 bucks.
But before ordering it i would like to make sure the problem is in the driver board and not the laser itself otherwise i will have to laser drivers and no laser to drive
But how do i test the laser without a driver?
Will connecting the laser directly to a 12V output from a computer PSU work? I mean computer electronics are sensitive as fuck so the PSU must have really clean smooth 12V output right?

>> No.1502160

>>1502114
yes, please blow your laser diode and start googling before asking

>> No.1502163

I'm looking to use H-bridge gate driver HIP4081A. I think it'll work for my application but I'm pressed for time and don't know if I can respin my board, so I'd appreciate a second set of eyes.

Possible issues:
1. My logic level is 3.3V. Datasheet specifies compatible with 5V, but the cutoff for a high logic input is 2.7V worst case. So I should be good, right?
2. Typical application schematics give 80V motor drive voltage. I'm using 12V. I don't see why that would cause issues but maybe something with the bootstrapping circuit? Just checking.
3. I don't see any information on bootstrap capacitor size in the datasheet so I'm going to just guess 1uF is fine. Thoughts?

>> No.1502199
File: 134 KB, 926x1014, Clipboard01.jpg [View same] [iqdb] [saucenao] [google]
1502199

How do i know if i have the voltage output type or the current output type?
I have the -000 version.
Chinks were no help. They have it listed as humidity sensor under a temperature sensor category.
I measured the resistance between the contacts on the jack and it's 105 ohms. But i am not sure if that is a resistor in there or just a transformer coil winding.

>> No.1502208

>>1502199
>000
it's in the table, baka
0-100A, 0-50mA

>> No.1502209

>>1502208
Except it's not, it could be both the 000 and 000v that is the problem, the description of the one i have is vague

>> No.1502220

>>1502209
in any case R = E/I. to get 1V drop across for 50mA through, you would add a 20 ohm resistor. pretty sure you're looking at 105 ohms coil resistance
also it would be fairly surprising for them to have not placed a V mark on it if it were a V type

>> No.1502225

>>1502209
Find out on your own. If it zaps you it's the current version.

>> No.1502231

>>1502220
yeah that seems right, i just don't want to fry my overpriced raspberry with adc hat

>> No.1502236

>>1502231
I seriously hope you're not feeding ac into a dc input. please for the love of god find someone's project to copy

>> No.1502246

>>1502236
why would that be a problem? if you offset the ac by a voltage divider reference point, so instead of swinging between 3v and -3v it will be between 0 - 1.5 and 1.5 - 3 then it should be fine

>> No.1502248
File: 634 KB, 640x480, lighthouse_controller.webm [View same] [iqdb] [saucenao] [google]
1502248

I just finished putting this latest prototype together. It is a lighthouse controller circuit made with four CD4000 ICs; two CD4017 counters, single CD4027 JK-flip-flop and CD4093 Schmitt trigger NAND. And a lot of diodes!

The light pattern can be changed with 16 DIP switches. Currently it just flashes an LED but I might add some kind of driver for driving a small incandescent bulb. It would also be nice to have a dark-activated switch in the circuit.

>> No.1502251
File: 2.45 MB, 1366x938, lighthouse_controller.png [View same] [iqdb] [saucenao] [google]
1502251

>>1502248
Schematic

>> No.1502256

>>1502248
congrats, you could do this in 5 minutes with fartduino for $2

>> No.1502268

>>1502248
bretty gud
but you should add some weak pulls to those diode-ORed lines

>> No.1502269

>>1498817
Anyone got a sty file for >>1498580 to share?
I have some documentation to do and would like it to look somewhat professional.

>> No.1502274

Like 8 years ago I had an old RC car charger power supply blow on me
I got a new one but I took it apart and showed it to an electronics guy to see if he knew what blew and he couldn't tell what it used to be.
Basically in the spot there were 2 legs still soldered to the board but the thing it was a part of burned away. The power supply worked when he shorted the gap but I've always wondered what the part actually was.

On top of the prongs there were like 2 small metal triangles if I remember correctly which makes me think it's a capacitor or something.
Fuck I will try to look for it, I've wanted to find out what it was for 8 years but kept forgetting about /ohm/

>> No.1502367
File: 76 KB, 1043x697, Untitled.png [View same] [iqdb] [saucenao] [google]
1502367

Will it work?

>> No.1502394

>>1502367
No, get yourself a 48V dimmer. (Or mod this one)

>> No.1502419

Just saw the latest Kreosan video, looks like something like a tesla coil but the circuit is a bit odd looking with what looks to be an SCR in series with mains. Anyone recognise the circuit?

>> No.1502426
File: 81 KB, 1043x697, Untitled.png [View same] [iqdb] [saucenao] [google]
1502426

>>1502394 48v dimmers are rare and modding is too much hassle (replacing transistors, drivers, aux power supply)
Could you please elaborate what will fail in such circuit? Cause i dont see any issues. 24v won't lit the leds and there wont be any over or reverse voltage conditions on the dimmer terminals.

>> No.1502432

>>1502426
Like this it might work, considering that power supplies are isolated from each other.
Dunno, I'd modify the dimmer. I guess if you have extra dimmers laying around - you can try it.

>> No.1502566
File: 99 KB, 1331x681, via_hole.png [View same] [iqdb] [saucenao] [google]
1502566

In KiCAD, I have what should be a 1 mil drill, 2 mil pad via. When I look at it in the 3D viewer, it has a giant hole in the middle. Did I screw something up? It's really a giant hole.

>>1502163
Also pls respond

>> No.1502589
File: 993 KB, 907x851, IMG_20181122_205255076.jpg [View same] [iqdb] [saucenao] [google]
1502589

I have this 1500W heater that has been giving me trouble for months
>plug got so hot ruined the wall outlet
>replaced the outlet and plug
>new plug melted
>replaced good portion of cable with orange extension and "heavy duty" plug
>fucking thing keeps melting the wirenuts
I'm not sure what I need to do. Do I need a heavier wirenut? Should I take the black cord off the heater completely and just wire the extension straight to the heater?

>> No.1502591

>>1502589
You should stop before you go all Beverly Hills Supper Club on your house

>> No.1502592
File: 2.83 MB, 3056x3258, IMG_20181122_210108460.jpg [View same] [iqdb] [saucenao] [google]
1502592

>>1502589
I should mention I just cut the cord because it wouldn't turn on and seems like the wires melted together (one wirenut melted completely and I haven't had a chance to replace yet).
Also, the power switch broke, it just rocks without clicking, how safe is this repair? Been going good for a few months this way.

>> No.1502593

>>1502591
No, its OK, the wirenut part is on concrete so it shouldn't catch anything on fire...Unless the outlet starts smoking again, but it hasn't got hot there since I replaced the cord and plug.

>> No.1502594

>>1502589
>wirenuts
Well there's your problem. Crimp some ferrules on those wires and put them in proper brass screw-terminals, and while you're at it make sure all the wire is of a good enough gauge.

>> No.1502595
File: 26 KB, 500x375, Bjork dude.jpg [View same] [iqdb] [saucenao] [google]
1502595

>>1502593
bro no

>> No.1502622
File: 9 KB, 480x360, 1542872753019.jpg [View same] [iqdb] [saucenao] [google]
1502622

Does anyone know a library for using an MPU9255 accelerometer (I2C) with an ESP32 using arduino IDE. I've got it showing up on the I2C scanner.

>> No.1502626

thinking about converting a keyboard I have with a Micro A (shitty) connector to USB C
I'm concerned about the polarity of the data lines, how would I hook them up to the keyboard to properly connect it?

>> No.1502644

>>1502622
http://lmgtfy.com/?q=MPU9255+arduino

>> No.1502649

>>1502626
If the data lines are the wrong way around you'll just show an error of nothing being connected and nothing bad will happen.

>> No.1502652

>>1502649
yes but it's a reversible connector

>> No.1502667
File: 5 KB, 608x504, lamp amp.png [View same] [iqdb] [saucenao] [google]
1502667

>>1502269
then use FrameMaker, not Word

>>1502426
then add your own driver on the output. I just put Pic related in my ham shack to get more light across the room without pulling much PWM current through my line of sight

>>1502566
your layout looks fine, probably a 3D viewer bug

>>1502163
possibly, as long as the worst case Voh(min) for what's driving it includes a decent margin over 2.7V and your trace is relatively short and not adjacent to any power traces
>bootstrapping at 12V
should work fine
>bootstrap cap size
10nF is typical

>>1502626
you need to read the USB-C standard because clearly you have no idea what you're doing

>> No.1502668

>>1502667
>you need to read the USB-C standard because clearly you have no idea what you're doing
I don't know what I'm doing yes

>> No.1502692

Anyone has experience in reusing DVD drive parts? I`m thinking of making a laser engraver to burn future PCBs. But most videos I find on the net end up with indian guys breaking the laser part and buying one from ali instead of using the infrared burner on the R/W head.

Another question, what would be a good way to transform a sinusoid-ish signal into a dc one in a linear way ? It`s for a sensor, the sensor works with magnetic coupling and feeding a highfrequency sinusoid to a uC won`t do much.

>> No.1502694

>>1502692
Is the laser from a DVD drive really enough to make a mark on a PCB? AFAIK the aluminium in a DVD is comparatively quite thin.

If a bridge rectifier won't work, then a PLL and/or ring mixer could be what you're looking for, multiply a sine by another of the same frequency and you should get a DC component, though you'd need some sort of phase locking.

>> No.1502697

>>1502694
>>1502694
The problem with the bridge is the voltage drop, the signal will barely go over the diode foward drop.
>laser
Yes it can, atleast the infre red one. They are class 1 because they are in a enclosure, but they are class 3 lazors

>> No.1502700

>>1502697
Could throw it into an op-amp to boost its amplitude, or use an op-amp ideal rectifier.

>> No.1502701

>>1502700
>op-amp ideal rectifier
thanks

>> No.1502704

>>1502694
does the DVD writer even ablate the aluminium? I would imagine thermal methods would just cause gas bubbles to form uncontrollably in the polycarbonate. CD writers just darken a dye spot to attenuate the reflection

>> No.1502708

>>1502701
Make sure to put an appropriately sized resistor across the filtration cap to ensure the voltage can drop properly. Ensure current can't go out of the capacitor into the op-amp either, if this circuit doesn't allow for such a thing then a MOSFET ideal rectifier will work. But if you want to have a quickly changing amplitude then it's likely that it's impossible to get the right resistor value on your capacitor, as too much will prevent it from discharging enough for the next cycle but not enough will not flatten the waveform enough. If this is the case, then you'd be better off with a sample+hold circuit that triggers on the peaks, if such a thing is possible at the sine frequencies you're expecting. The mixer/PLL method may also be an option, but I don't know how PLLs work.

>> No.1502709

>>1502704
Well it can't ablate it anyhow since a DVD's foil is protected on both sides by a layer of plastic. I was more assuming the heat caused it to wrinkle up or something, or perhaps the plastic right above it chars. I'm no expert, but either of these would take much less power than what's required to cut through thin metal on a laser cutter. Divide that by ten to simply engrave it, and maybe. Perhaps such a laser might be a neat way to get rid of areas of photoresist on a PCB for high-precision etching.

>> No.1502714

So basically i have an arduino which monitors a probe and is saving readings from this probe to eeproom (pernament memory). It saves the readings about once every two seconds.
To mi absolute horror i recently found out that even though it is
>2018
they use some shitty technology that only allows about 10 000 - 100 000 writes to the eproom before it's toast
this means the arduino will become useless in mere weeks.
So what is a good alternative to this?
I don't want to use an SD card module because that is needlessly expensive and massive, i only need to save a single number

>> No.1502720

>>1502714
>expensive
Literally less than 50c each for the module, perhaps $2 for the SD card. But something like the w25Q16bvsig modules out there might be what you're looking for, or anything in the non-volatile flash memory section of digikey if you're not afraid of a little SMT soldering.

>> No.1502723
File: 68 KB, 640x640, 2516-memory-module-W25Q16BVSIG-serial-SPI-flash-memory-16M-BIT.jpg_640x640[1].jpg [View same] [iqdb] [saucenao] [google]
1502723

>>1502720
>SMT soldering.
Not happening. But they sell them chinkified so that is not a problem.
So it has unlimited amount of writes and doesn't need power to remember the saved data?
That would be perfect, i just pray to god there is an arduino library for it, since i sure as fuck ain't bit banging that shit

>> No.1502725

>>1502723
I don't know if it is "unlimited" writes, but the ali description didn't have "eeprom" in the title. Rule 2 applies. You could always use a quartz-window DIP EPROM with a UV LED hooked up to your circuit to wipe itself whenever you need more storage. It would be quite a unique method, but I've no idea if those EPROMs are any better cycle-wise than normal EEPROM, probably not.

>> No.1502729
File: 2.75 MB, 263x261, WTF.gif [View same] [iqdb] [saucenao] [google]
1502729

>>1502589
It's probably the heater that's malfunctioning, not the cables if it's drawing enough current to melt the cables.

>> No.1502732

>>1502723
W25Q16BV is rated 100 000 write cycles if you're going over the same location over and over again. If you're storing 256 bytes or less you can make a circular buffer using one page at a time and you can multiply that cycle count by page count, would give you ~800 000 000 cycles total

If you don't want to do that circular wear levelling you could use FRAM instead, non volatile, no pages to worry about, rated cycles in the trillions +

>> No.1502737

>>1502709
pretty sure it's photosensitive dye
>Perhaps such a laser might be a neat way to get rid of areas of photoresist on a PCB for high-precision etching
depending on kerf, yes, but undercut during the etch process is gonna start to hurt you before you can get much below 6/6 rule

>>1502714
is there any reason you wouldn't want to use battery-backed RAM?
also, don't use flash like those 25xx series parts. endurance applies to whole pages on those, and isn't all that great. instead, use a serial EEPROM where you can individually erase and write single addresses. with an oldskool 93LC66A, you have 512 bytes available. you can use a single 3-byte block until it reaches 1 million, then consider it "worn out" and move on to the next one. you could use the first byte as a pointer to which counter is current (and also use as a millions digit), or just search backwards upon reset to find the first programmed bytes, or just keep 170 counters in parallel which you would add all up when reporting. a naive implementation could count to 170 million or so before running out of writes, or (if Microchip Total Endurance software understands me) last about 5 years before 1ppm failure

>>1502732
fwiw flash is about 100x slower to erase than the 93LCxx
also the 93LCxx might can still be gotten in DIP

>> No.1502745

>>1502644
I've already googled it. And it's not arduino, it's ESP32

>> No.1502753

>>1502732
So let's say you want to rotate the blocks you use for saving.
I only need to save an 8 digit number so i have lot's of space to move it around.
So you save it say say 100 block offset from 0 so you aren't using the same blocks every time and then arduino reboots.
How will your code now that the current data is saved at the 100 offset position wheen you have nowhere to save this position between reboots?

>> No.1502755

>>1502732
>FRAM
holy shit i just checked, that ain't cheap, that ain't cheap at all, in fact a single chip costs more than the whole arduino

>> No.1502770

>>1502160
how about attaching it to a buck converter with voltage set to 1v and then slowly increase until i see light? i do not need to cut anything, i literally just need to see the diode is capable of emitting light and thus not damaged. i would turn it off right after

>> No.1502832

>>1501723
>>1501861

I've never soldered SMD, you think it's worth learning early on?


>>1501563
the fcc does drop by your house. My uncle used to do that.

>> No.1502841

I`m going tobuy a scope this black friday, is 600Mhz, 1Gsa and 40 kpts good? How this compares to the rigol Ds1102e ? Found these at the same price.

>> No.1502844

>>1502841
Fug, the site was wrong, it`s a 60Mhz one. Fucking typos

>> No.1502848

>>1502770
At a minimum, you need a constant-current drive. For sustained use, you need to split off part of the beam and vary the drive current to maintain constant output power, but you can probably skip that for a brief test at minimum power.

The main issue with high-power laser diodes (>5-10mW) is a combination of three things:
1. The maximum current isn't much higher than the threshold current (the minimum current at which the diode produces a coherent beam).
2. Both the maximum current and threshold current vary with die temperature. The maximum current for a warm die will be lower than the threshold current for a cool die, so you have to use the measured output power in the control loop.
3. Exceeding the maximum power by even a relatively small amount can damage the diode in less than a microsecond.

The last one means that you have to be particularly careful of power-on transients.

>> No.1502851

>>1502268
Seems like I forgot to add them to the schematic. I used 100 k pull-down resistors on 4017 reset pins.

>>1502256
Yeah, but what's the fun with that?

>> No.1502862

>>1502848
educational post, thanks

>> No.1502870

>>1502848
So I'm basically fucked then.
I have to order a new driver board and simply risk ending up with two useless drivers for a broken laser.

>> No.1502875

>>1502870
Although I don't think it's the laser, when i was trying to use it for the second time, it did't work, then turned on for like a second and off again and then didn't work since
Welp guess i am getting a new driver, just checked and it costs $20. Kind of pricey but not too bad, i won't be too sad to toss it out if the laser turns out not to be working anyway

>> No.1502943

sorry if stupid question, but how much of a gap is there really from hobbyist/amateur electronics to designing custom computer hardware or networking equipment? To put in another way, how long would it take for someone motivated enough to go from designing some simple circuits to creating your own networking switch or embedded computer?

>> No.1502972
File: 453 KB, 406x662, Parachute Recovery.png [View same] [iqdb] [saucenao] [google]
1502972

I am designing a parachute recovery system for a hobby water rocket.

I want to the nose cone to be held in tension by a rubber band attached to a servo arm. After a fixed period of time (determined by experimental launches to time apogee), I want the servo arm to be given power and turn 90 degrees, thus allowing the rubber band to slip off of the arm, letting the nose cone detach from the rocket, thus pulling the parachute out of the rocket body. See below link to visualize this process.

What electronic chip can I interface with the servo arm to accomplish this task? How would I power the chip/servo arm?

I think I could accomplish this idea with an Arduino Nano, but I am looking for something lighter and smaller.

>> No.1502976

>>1502943
I started designing circuits my second year of college, was probably capable of designing an embedded computer in my fourth year... so about 2 years, let's say 20 hours a week because you're skipping peripheral courses like math.

>> No.1502980

>>1502976
that sounds sooner than I thought! thanks anon!

>> No.1503033
File: 37 KB, 400x496, mfw.jpg [View same] [iqdb] [saucenao] [google]
1503033

>>1502745
>it's not arduino
>using arduino IDE

>> No.1503041

test

>> No.1503060

>>1502832
>worth it early on
ehhhhh, maybe. there is a lot of value to be had in sticking components into solderless breadboards, which don't work so well with SMD
do consider chinkshit through-hole component collections though. you can probably save quite a bit of cash over buying from a reputable big-name global dealer

>>1502851
>100k pull downs
am proud

>> No.1503064

Anyone knows of a analog circuit that can calculate the average of a signal during a certain ammount of time? Or the integral. (Over the period of something)

>> No.1503069

>>1503064
Or a way to generate sinusoids with a cheap uC and normal components, I need to estimulate a coupled pair of coils to a certain frequency and see the response, I tried making a Armstrong oscillator but I must have fucked it up because it doesn`t oscilate. (The coils have little inductance, I have no way to measure it but 80-160 turns, air core 32mm diameter, 3 and 6 cm long. Maybe that has something to do with it.)

>> No.1503094
File: 51 KB, 1100x1100, 51NdEBI1aGL._SL1100_.jpg [View same] [iqdb] [saucenao] [google]
1503094

What is generally a safe operating temp for transistors?
I have a D1047, it says Tj 150°C, is it safe to put the temp warning on the heatsink to 70° and make it shut down at 90°?

>> No.1503096
File: 6 KB, 610x380, RLC-osc.png [View same] [iqdb] [saucenao] [google]
1503096

>>1503069
Bookmark http://electronbunker.ca/eb/InductanceCalc.html
80-32-30 about 145µH and 160-32-60 about 350µH both wound with 0.3mm wire
The emitter-coupled oscillator always works (1.5V, current sets loop gain via R)

>> No.1503245
File: 5 KB, 400x204, 1531831635055.gif [View same] [iqdb] [saucenao] [google]
1503245

>>1503064
sure, and cleverly enough it's called an integrator, Pic related

>>1503096
aka Peltz oscillator

>> No.1503251

>>1503096
This circuit looks amazing. Would it be possible to use one (with RF transistors) to make a 25MHz sine to feed my up-converter? It would certainly save me from having to filter a crystal's square wave.

>> No.1503254

What does it mean that arduino adc has 10 bit resolution?

>> No.1503257

>>1503251
Not in a way that'd give reasonable frequency stability.

>>1503254
Means the ADC's output has 2^10 possible values, 0 to 1023.

>> No.1503258

>>1503254
It means it can only measure the voltage at so many discrete levels. The number of different voltages it can measure is 2^n, where n is the number of bits. 10bit = 1024 different values. An ADC uses a reference voltage, one of which I think is 1V or so, meaning if you're measuring a voltage and you ADC has a range from 0V to 1V, the voltage will be measured in 976.5µV steps [ (Vcc-Vee) / 2^n ]. So if you tried to measure 0.35565V, it would be rounded to the nearest 0.0009765V, which is 0.35547V. Naturally, changing your reference voltage will change this value. As far as I know, the analog_read function will return an integer from 0 to 1023, so I think that example would return the value 364, give or take 1. So to get the actual voltage in software you'd need to multiply that by the reference voltage over the number of possible values, which is 1/1024 in this case.
t. never used an ADC

>> No.1503280

>>1503258
Thanks. So if i want higher precision, i need to use a smaller reference voltage range, since 1V - 2V can be split into smaller 1024 pieces than say 0V - 5V

>> No.1503292

>>1503258
>>1503257
I have some PCF 8591 4-ch A/D-converter chips. It has 8 bits resolution. Do you think the resolution would be okay for building a weather station?
ch1: temp sensor (NTC thermistor)
ch2: light sensor (LED sensor)
ch3: humidity sensor
ch4: ?

>> No.1503294
File: 110 KB, 1641x1212, Clipboard02.jpg [View same] [iqdb] [saucenao] [google]
1503294

>>1503258
>>1503280
Ok so basically I have a CT sensor that generates AC output in miliamps.
The 33ohm resistor is used so that the ct sensor generates maximum possible voltage swings between -2.5v and 2.5v at its max output of about 70ma
This is because arduino can¨t handle negative voltage, so i have to reference the AC against a 2.5V ground base, so that way on the analog pin it swings between 0 - 5V (0 - 2.5V for the top of the sine wave and vice versa)
But this means i am only getting half resolution because i have to waste half of the voltage on counteracting the negative part of the sine wave.
Pic related is the official guide how to connect the CT to arduino.
So i was thinking (my changes are marked in green crayon) what if i add a simple diode to remove the negative half of the sine and then i double the resistor value so i get swings from the ct between 0 - 5V (i don't need the bottom of the sine, it's same as the top anyway), then i will effectively double the precision, would that work?

>> No.1503295

>>1503245
How would I limit the integration to a certain time?

>> No.1503297

>>1503294
wait i just realized a simple diode won't work, the CT will go ape shit and generate massive voltage and burn the diode
doesn't matter, i can just do some diode fuckery to shortcircuit the ct output during negative sine and work normally during positive part of sine

>> No.1503308
File: 3 KB, 512x182, 1532610482519.png [View same] [iqdb] [saucenao] [google]
1503308

>>1503297
you should consider a full-wave ideal rectifier made of op amps

>>1503295
cutting off the input and draining the cap, e.g. with CD4066-style CMOS switches

>>1503292
that's a design decision you need to make. do the math

>>1503297
sort of, the MCU's absolute limit on input voltages at the low end is -0.3V
if you were to insert a Schottky diode in reverse (for lower Vf drop), then take that -0.5 to 5V half-clipped sine and scale it down with a voltage divider in the several to tens of kilohms range, you could get the max voltage to 1.1V to match the internal reference, and the clipped bottom half wouldn't exceed 100mV or so, which might be okay

>> No.1503324

>>1503308
I found out that chinks sell 16bit adc chinkverter for just buck so i will be getting that since that will have more precision than i will ever need

>> No.1503367

>>1503324
but you still have to know:
is it fast enough?
is it accurate enough?
does it accept negative voltages gracefully?

>> No.1503370

is it save to stick the probes on this thing into an electrical outlet with 230V AC?
https://www.amazon.com/Owon-VDS1022I-USB-Oscilloscope-Isolation/dp/B00HC4UP52

>> No.1503374

>>1502667
>then use FrameMaker, not Word
Both the premise and the answer are very, very wrong.

>> No.1503375

>>1503324
I am sure it will be fine, chinks write it can do 800sps which i assume is samples per second and i only need to sample 50hz which is a snail pace

>> No.1503376

>>1503367
>>1503375

>> No.1503379

>>1502972
Use a monostable multivibrator, just 2 transistors and some bits and bobs. Power comes from a capacitor. Drop the servo and use an electromagnet instead.

>> No.1503396

>>1503370
Customer Questions & Answers

>> No.1503398

>>1503396
nice i didn't notice that
i just feels really scary to be connecting an usb device to 230v when it's connected to my house tier price computer

>> No.1503422

>>1502972
>lighter
>smaller
digispark?

>> No.1503433

what material do you use to insulate coils? Like a varnish or something.

>> No.1503451

what are some good batteries to use for long term low consumption projects?
for example ardiuno mini that wakes up from time to time and does shit
i don't really want to use ordinary aa batteries as i bet there are much better ones with better mah/$ ratio
they don't have to be rechargeable

>> No.1503458

>>1503433
for what, from what, at what voltage?
urethane is common

>>1503451
>does shit
not worth asking what kind of "shit" you'll be doing
anyway lithium-iron disulfide batteries are pretty easy to get ("1.5V lithium") and seem about right for that sort of project. enjoy
https://en.wikipedia.org/wiki/Comparison_of_commercial_battery_types

>> No.1503463

>>1503451
Nickel Iron

>> No.1503467

NEW BREAD
>>1503466
>>1503466
>>1503466
NEW BREAD

>> No.1503470

>>1503465
you had ONE job

>> No.1503474

>>1503458
0-30cm water, 12v max.

>> No.1503483

>>1503474
for example, this seems like some pretty primo shit
http://www.industrialpolymers.com/products/encapsulates-potting-compounds

>> No.1504083

>>1502592
My nigger-rigging here has started to fail after months. I suspect that some corrosion is building up on the bridge and stopping the electricity. The only way I can get it to work is if I tap the wire with something non-conductive. What is a better option for repair, other than replacing the switch with another switch?